Download as pdf or txt
Download as pdf or txt
You are on page 1of 54

LE 1.

04 HEMODYNAMIC DISORDERS
September 12, 2022
● Exudates accumulate due to increase in vascular permeability
OUTLINE caused by inflammatory mediators.
I. Edema and Effusions ● Specific Gravity of > 1.012
II. Hyperemia and Congestion ● Protein content > 3g/dl
III. Hemostasis, Hemorrhagic Disorders, and Thrombosis
A. Normal Hemostasis TRANSUDATE
B. Hemorrhagic Disorders ● Protein poor
C. Thrombosis ● Non-inflammatory edema and effusions which are common in many
D. Disseminated Intravascular Coagulation (DIC) disorders, including heart failure, liver failure, renal disease, and
IV. Embolism malnutrition
A. Pulmonary Embolism ● Specific Gravity of < 1.012
B. Systemic Thromboembolism ● Protein content < 3 g/dl
C. Fat Embolism
D. Air Embolism INCREASED HYDROSTATIC PRESSURE
E. Amniotic Fluid Embolism
V. Infarction ● Increased hydrostatic pressure is mainly caused by disorders that
VI. Shock impair venous return
A. Pathogenesis of Septic Shock ● If the impairment is localized then the resulting edema is confined to
B. Stages of Shock the affected part.
C. Clinical Course ○ Ex. deep venous thrombosis [DVT] in a lower extremity
LEGEND REDUCED PLASMA OSMOTIC PRESSURE
Remember Lecturer Book Presentation
● Under normal circumstances albumin accounts for almost half of the
total plasma protein; it follows that conditions leading to inadequate
synthesis or increased loss of albumin from the circulation are
I. EDEMA AND EFFUSIONS common causes of reduced plasma oncotic pressure
● Disorders that perturb cardiovascular, renal, or hepatic function ● Reduced albumin synthesis occurs mainly in severe liver disease
● Edema fluids and effusions may be Inflammatory or Non- and protein malnutrition.
inflammatory ● Reduced plasma osmotic pressure leads in a stepwise fashion to
● Edema edema, reduced intravascular volume, renal hypoperfusion, and
○ Accumulation of fluid in the tissues secondary hyperaldosteronism.
● Effusions ● Nephrotic syndrome
○ Accumulation of fluid in the body cavities ○ In which albumin leaks into the urine through abnormally
● Hydrostatic pressure permeable glomerular capillaries.
○ Pushes water and salts out of capillaries into the interstitial space
● Plasma colloid osmotic pressure SODIUM AND WATER RETENTION
○ Pulls water and salts back into the vessel.
● Increased salt retention—with obligate retention of associated
water—causes both increased hydrostatic pressure (due to
intravascular fluid volume expansion) and diminished vascular
colloid osmotic pressure (due to dilution).
● Salt retention occurs whenever renal function is compromised, such
as in primary kidney disorders and in cardiovascular disorders that
decrease renal perfusion.

FIgure 1. Factors influencing fluid movement across capillary walls

● Normally, hydrostatic and osmotic forces are nearly balanced so that


there is little net movement of fluid out of vessels.
● Elevated hydrostatic pressure or diminished colloid osmotic pressure
disrupts this balance and results in increased movement of fluid out
of vessels FIgure 2. Mechanisms of systemic edema in heart failure, renal failure,
● If the net rate of fluid movement exceeds the rate of lymphatic malnutrition, hepatic failure, and nephrotic syndrome.
drainage, fluid accumulates.
● Within tissues the result is edema, and if a serosal surface is ● One of the most important causes of renal hypoperfusion is
involved, fluid may accumulate within the adjacent body cavity as an congestive heart failure, which (like hypoproteinemia) results in the
effusion activation of the renin-angiotensin-aldosterone axis
● In early heart failure, this response is beneficial, as the retention of
EXUDATE sodium and water and other adaptations, including increased
● Protein rich (EXcessive protein content) vascular tone and elevated levels of antidiuretic hormone, improve
● Inflammation related edema and effusions cardiac output and restore normal renal perfusion.

1
● However, as heart failure worsens and cardiac output diminishes,
the retained fluid merely increases the hydrostatic pressure, leading ■ The lungs are often two to three times their normal weight,
to edema and effusions. and sectioning yields frothy, blood-tinged fluid—a mixture
of air, edema, and extravasated red cells
LYMPHATIC OBSTRUCTION ■ Pulmonary edema is a common clinical problem that is
most frequently seen in the setting of left ventricular failure;
● Trauma, fibrosis, invasive tumors, and infectious agents can all
it can also occur with renal failure, acute respiratory
disrupt lymphatic vessels and impair the clearance of interstitial fluid,
distress syndrome (ARS) and pulmonary inflammation or
resulting in lymphedema in the affected part of the body.
infection.
● Filariasis,
■ Edema in the pulmonary interstitium and the alveolar
○ Parasitic organism in which the organism induces obstructive
spaces impedes gas exchange (leading to hypoxemia)
fibrosis of lymphatic channels and lymph nodes
and also creates a favorable environment for bacterial
○ This may result in edema of the external genitalia and lower limbs
infection.
that is so massive as to earn the appellation elephantiasis
■ Pulmonary edema is often exacerbated by pleural
effusions, which may further compromise gas exchange by
compressing the underlying pulmonary parenchyma
○ Brain edema
■ Localized or generalized depending on the nature and
extent of the pathologic process or injury. The swollen brain
exhibits narrowed sulci and distended gyri, which are
compressed by the unyielding skull.
■ Brain edema is life threatening; if severe, brain substance
can herniate (extrude) through the foramen magnum, or the
brainstem vascular supply can be compressed. Either
condition can injure the medullary centers and cause death
○ Pleural effusions
■ Hydrothorax- Pleural Cavity
■ Hydropericardium - Pericardial Cavity
■ Hydroperitoneum or Ascites - Peritoneal Cavity
○ Peritoneal effusions (ascites)
■ Resulting most commonly from portal hypertension are
prone to seeding by bacteria, leading to serious and
sometimes fatal infections
○ Transudative effusions
■ Are typically protein-poor, translucent, and straw colored
■ An exception are peritoneal effusions caused by lymphatic
blockage (chylous effusion), which may be milky due to
the presence of lipids absorbed from the gut.
○ Exudative effusions
■ Are protein-rich and often cloudy due to the presence of
white cells

Figure 3. Pathophysiologic Categories of Edema FIgure 4. Subcutaneous Edema with Pitting Edema

MORPHOLOGY
○ Subcutaneous edema
■ Diffused or more conspicuous in regions with high hydrostatic
pressures. Its distribution is often influenced by gravity
■ Subcutaneous edema is important primarily because it signals
potential underlying cardiac or renal disease; however, when
significant, it can also impair wound healing and the clearance
of infections
○ Pitting edema
■ Finger pressure over markedly edematous subcutaneous
tissue displaces the interstitial fluid and leaves a depression,
a sign called pitting edema. FIgure 5. Periorbital Edema
○ Periorbital edema
■ Edema resulting from renal dysfunction often appears initially
in parts of the body containing loose connective tissue, such
as the eyelids; periorbital edema is thus a characteristic
finding in severe renal disease
○ Pulmonary edema

2
FIgure 10. Brain Edema

FIgure 6. Pulmonary Edema

FIgure 11. l Transudative Effusion Left, Exudative Effusion Right

FIgure 7.Histology of Normal lung tissue

FIgure 8. Histology of Lung with Pulmonary Edema FIgure 12. Chest X-ray Showing Pleural Effusion

II. HYPEREMIA AND CONGESTION


HYPEREMIA
● Is an active process in which arteriolar dilation leads to increased
blood flow
○ Ex. At sites of inflammation or in skeletal muscle during exercise
● Affected tissues turn red (erythema) because of increased delivery
of oxygenated blood.
CONGESTION
● Is a passive process resulting from reduced venous outflow of blood
from a tissue.
○ Systemic ( Congestive Heart Failure)
○ Local ( Isolated venous obstruction)
● Congested tissues have an abnormal blue-red color (cyanosis) that
stems from the accumulation of deoxygenated hemoglobin in the
FIgure 9. Normal Brain of Dead Specimen affected area.

3
MORPHOLOGY
○ Active Hyperemia
■ Swollen arteries and arterioles are full of RBC
○ Congestion/Chronic Passive Congestion
■ Dilated veins or venules full of RBC; hemosiderin-laden
macrophages
○ Acute pulmonary congestion
■ Is marked by engorged alveolar capillaries, alveolar septal
edema, and focal intra-alveolar hemorrhage.
○ Chronic pulmonary congestion
■ Often caused by congestive heart failure, the septa are
thickened and fibrotic, and the alveoli often contain
numerous macrophages laden with hemosiderin (heart
failure cells) derived from phagocytosed red cells Figure 16.Heart failure
■ Dilated veins or venules full of RBC; hemosiderin-laden
macrophages
○ Acute hepatic congestion
■ The central vein and sinusoids are distended. Because the
centrilobular area is at the distal end of the hepatic blood
supply, centrilobular hepatocytes may undergo ischemic
necrosis, and the periportal hepatocytes—better
oxygenated because of proximity to hepatic
arterioles—may only develop fatty change.
○ Chronic passive hepatic congestion
■ The centrilobular regions are grossly red brown and slightly
depressed (because of cell death) and are accentuated
against the surrounding zones of uncongested tan liver
(nutmeg liver)

FIgure 17.Acute Hepatic Congestion

FIgure 13.Active Hyperemia


FIgure 18.Chronic Passive Hepatic Congestion

III. HEMOSTASIS, HEMORRHAGIC DISORDERS,


AND THROMBOSIS
A. NORMAL HEMOSTASIS
● Hemostasis
○ Process by which blood clots form at sites of traumatic vascular
injury, which prevent or limit the extent of bleeding
○ Transient reflex neurogenic arteriolar vasoconstriction augmented
by endothelin (potent endothelial-derived vasoconstrictor)
○ Platelet adhesion and activation (shape change and secretory
granule release) by binding to exposed subendothelial ECM
○ Secreted products recruit other platelets to form a temporary
hemostatic plug (primary hemostasis)
FIgure 14.Congestion/Chronic Passive Congestion ○ Activation of the coagulation cascade by release of tissue factor
(aka thromboplastin or factor III), a membrane-bound
lipoprotein procoagulant factor synthesized by endothelium
○ Coagulation culminates in thrombin generation and conversion of
circulating fibrinogen to insoluble fibrin
○ Thrombin also induces additional platelet recruitment and
granule release
○ Polymerized fibrin and platelet aggregates together form a solid,
permanent plug (secondary hemostasis)
○ Activation of counter-regulatory mechanisms (e.g., tissue
plasminogen activator [t-PA]) restricts the hemostatic plug to
the site of injury
● Hemorrhagic D.O
○ The hemostatic mechanisms are inadequate to prevent abnormal
bleeding.
FIgure 15. Acute pulmonary congestion leading to pulmonary edema ● Thrombotic states
○ Blood clot formation occurs within intact vessels

4
SEQUENCE OF EVENTS LEADING TO HEMOSTASIS AT A
SITE OF VASCULAR INJURY
1. Vasoconstriction
2. Primary hemostasis
3. Secondary hemostasis
4. Thrombus and antithrombotic events
ENDOTHELIUM
● Regulate several, frequently opposing aspects of hemostasis
● The balance between the anticoagulant and procoagulant activities
of endothelium often determines whether clot formation,
propagation, or dissolution occurs
● Normal endothelial cells express a multitude of factors that inhibit
the procoagulant activities of platelets and coagulation factors and
that augment fibrinolysis
● The antithrombotic properties of endothelium can be divided into
activities directed at platelets, coagulation factors, and fibrinolysis:
○ Platelet inhibitory effects
■ Intact endothelium blocks platelet access to the thrombogenic
subendothelial matrix.
■ PGI2 and nitric oxide (NO) inhibit platelet binding.
■ Adenosine diphosphatase degrades ADP, an inducer of
platelet aggregation FIgure 19. Platelet adhesion and aggregation. Von Willebrand factor functions as
○ Anticoagulant effects an adhesion bridge between subendothelial collagen and the glycoprotein Ib
■ Membrane-associated thrombomodulin converts thrombin to (GpIb) platelet receptor. Aggregation is accomplished by fibrinogen bridging
an anticoagulant protein. GpIIb-IIIa receptors on different platelets. Congenital deficiencies in the various
■ Activated protein C/protein S complex potently inhibits factors receptors or bridging molecules lead to the diseases indicated in the colored
Va and VIIIa. boxes. ADP, Adenosine diphosphate.
■ Heparin-like surface molecules facilitate plasma antithrombin
III inactivation of thrombin. COAGULATION CASCADE
■ Tissue factor pathway inhibitor (TFPI) blocks tissue
factor/factor VIIa complexes ● Each step in the cascade involves the following:
○ Fibrinolytic effects ○ Enzyme (activated coagulation factor)
■ t-PA cleaves plasminogen to form plasmin, which in turn ○ Substrate (inactive proenzyme form of a coagulation factor)
degrades fibrin ○ Cofactor (a reaction accelerator)
○ These are brought together on the negatively charged platelet
PLATELETS phospholipid surface. Assembly of the various complexes also
requires calcium, which binds to γ-carboxylated glutamic acid
● Play a critical role in hemostasis by forming the primary plug residues on factors II, VII, IX, and X.
● Disc-shaped anucleate cell fragments that are shed from ○ Clotting thus tends to remain localized to sites where assembly
megakaryocytes in the bone marrow into the bloodstream. can occur (e.g., surfaces of activated platelets or endothelium).
● Their function depends on several glycoprotein receptors, a ● The in vitro measurements:
contractile cytoskeleton, and two types of cytoplasmic granules ○ Prothrombin time (PT) - measured after the addition of tissue
● Platelet ECM adhesion factor, phospholipids, and calcium.
○ Mediated through von WIllebrand factor (vWF), acting as a bridge ■ It evaluates the function of extrinsic pathway proteins (VII, X, II,
between receptors (mostly GpIb) and exposed collagen. V, and fibrinogen).
○ Genetic deficiencies of vWF (von Willebrand Disease) or GpIb ○ Partial thromboplastin time (PTT) - measured after the addition
(Bernard-Soulier Syndrome) result in bleeding disorders. of negatively charged particles (e.g., ground glass);
● Platelets change shape ■ It screens for the function of intrinsic pathway proteins (XII, XI,
○ From smooth disks to spiky ovoids with markedly increased IX, VIII, X, V, II, and fibrinogen).
surface area. ● In vivo:
○ This change is accompanied by conformational changes in cell ○ Factor VIIa/tissue factor complexes are the important activators
surface GpIIb-IIIa changes that increase its affinity for fibrinogen of factor IX
and by the translocation of negatively charged phospholipids ○ Factor IXa/Factor VIIa complexes are the most important
(particularly phosphatidylserine) to the platelet surface. activators of factor X.
■ These phospholipids bind calcium and serve as nucleation
sites for the assembly of coagulation factor complexes. B. HEMORRHAGIC DISORDERS
● Platelet granule secretion (release reaction) ● Hemorrhage results from disorders of vessel walls, platelets, or
○ Occurs shortly after adhesion coagulation factors
○ Alpha granules express P-selectin adhesion molecules and ● The most common etiologies for a mild bleeding tendency are
contain coagulation and growth factors inherited defects in vWF, consumption of nonsteroidal
○ Dense bodies or Delta granules contain adenosine nucleotides antiinflammatory agents (e.g., aspirin), and renal failure (uremia)
■ ADP, calcium, and vasoactive amines (histamine) ● Defects in Primary hemostasis
● ADP is a potent mediator of platelet aggregation ○ Most commonly due to low platelet counts (thrombocytopenia)
(recruitment). ○ Abnormal platelet function, or Von Willebrand Disease
● Calcium is important for the coagulation cascade. ○ These manifest as minute, 1- to 2-mm petechial hemorrhages or
● Platelet aggregation slightly larger purpura in skin, mucous membranes, or serosal
○ Platelets adhering to other platelets surfaces.
○ Promoted by ADP and Thromboxane A 2 (TxA2) ● Defects in secondary hemostasis
● ADP activation ○ Involve coagulation factor abnormalities and typically present with
○ Changes platelet GpIIb-IIIa receptor conformation to allow bleeding into joints (hemarthrosis) or soft tissues.
fibrinogen binding ● Generalized defects involving small vessels:
○ Fibrinogen bridges multiple platelets forming a large aggregates ■ Bruises - manifest as >1- to 2-cm subcutaneous ecchymoses
■ GpIIb-IIIa deficiencies result in Glanzmann thrombasthenia ■ Hematoma- large amount of extravasated blood will generate a
bleeding disorder. palpable mass

5
CLINICAL SIGNIFICANCE OF HEMORRHAGE DEPENDS ○ Factor V gene mutations are the most common;
ON THE VOLUME AND RATE OF BLOOD LOSS ○ 2% to 15% of Caucasians (60% of patients with recurrent deep
● Rapid loss of less than 20% or slow losses of even larger amounts vein thrombosis) carry the so-called Leiden mutation, rendering
may have little impact; greater losses result in hemorrhagic factor V resistant to protein C inactivation.
(hypovolemic) shock ○ A single nucleotide change (G20210A) in the 3′ untranslated
● Location is also important: Bleeding that would be inconsequential in region of the prothrombin gene (1% to 2% of the population) leads
subcutaneous tissues may cause death in the brain to elevated prothrombin levels and a threefold increased risk of
● Chronic blood loss (e.g., peptic ulcer or menstrual bleeding) can venous thrombosis.
result in iron deficiency anemia

C. THROMBOSIS
● Primary abnormalities that lead to thrombosis:
1. Endothelial injury
2. Stasis or turbulent blood flow
3. Hypercoagulability of the blood (the so-called Virchow triad)
● Results from exposed subendothelial ECM, increased platelet
adhesion, elevated procoagulant production (tissue factor,
plasminogen activator inhibitor), or reduced anticoagulant activity
(PGI2, thrombomodulin, t-PA).
● Deficiencies of antithrombin III, protein C, or protein S also typically
present with venous thrombosis and recurrent thromboembolism.

Figure 20. The Virchow triad in thrombosis. Endothelial integrity is the most
important factor. Injury to endothelial cells can alter local blood flow and affect
coagulability. Abnormal blood flow (stasis or turbulence), in turn, can cause
endothelial injury. These factors may promote thrombosis independently or in
combination.

ENDOTHELIAL INJURY
● Dominant and can independently cause thrombosis (e.g.,
Figure 21. Hypercoagulable States
endocarditis or ulcerated atherosclerotic plaque).
● Can be due to hemodynamic stresses (e.g., hypertension or HEPARIN-INDUCED THROMBOCYTOPENIA (HIT)
turbulent flow), endotoxin, radiation, or noxious agents
SYNDROME
(homocystinuria, hypercholesterolemia, or cigarette smoke).
● A serious, potentially life-threatening disorder that occurs following
ALTERATIONS IN NORMAL BLOOD FLOW the administration of unfractionated heparin
● Results from the formation of antibodies that recognize complexes of
● Normal blood flow is laminar (i.e., cellular elements flow centrally in
heparin and PF4 on the surface of platelets, complexes of
the vessel lumen, separated from endothelium by a plasma clear
heparin-like molecules and PF4-like proteins on endothelial cells.
zone)
● PF4 protein
● Stasis and turbulence:
○ Normally found in platelet alpha granules and is released on
○ Disrupt laminar flow and bring platelets into contact with the
activation of platelets.
endothelium.
○ Released PF4 binds to heparin and undergoes a conformational
○ Prevent dilution of activated clotting factors by flowing blood.
change that results in the formation of a neoantigen against which
○ Retard the inflow of clotting inhibitors.
IgG antibodies are formed.
○ Promote EC activation.
○ PF4-IgG immune complex attaches to and cross-links the Fc
● Stasis causes thrombosis in the venous circulation, cardiac
receptors on the platelet surface, which leads to platelet activation
chambers, and arterial aneurysms; turbulence causes thrombosis
and aggregation.
in the arterial circulation as well as endothelial injury.
○ Platelet activation results in the release of more PF4, creating
● Hyperviscosity syndromes (e.g., polycythemia) or deformed
more target antigen for HIT antibodies.
erythrocytes (e.g., sickle cell anemia) result in small vessel stasis
○ Prothrombotic state - augmented by activation of endothelium
and also predispose to thrombosis.
by binding of HIT antibodies to PF4-like proteins on their surface.
HYPERCOAGULABILITY ○ Thrombocytopenia state - binding of HIT antibodies to platelets
results in their removal by macrophages
● Refers to an abnormally high tendency of the blood to clot, and is ○ Thrombocytopenia is the most common manifestation,
typically caused by alterations in coagulation factors. thrombosis is the most serious complication.
○ Has a particularly important role in venous thrombosis and can be ○ It occurs in approximately 50% of cases and affects both veins
divided into primary (genetic) and secondary (acquired) disorders and arteries. Necrosis of the skin, gangrene of the limbs, stroke,
● Heritable hypercoagulable states: and myocardial infarction are some of the sequelae.

6
○ Diagnosis requires the demonstration of anti–PF4-heparin FATE OF THE THROMBUS
antibodies. Low-molecular weight heparin preparations induce
HIT less frequently ● If a patient survives the immediate effects of a thrombus, some
combination of the following occurs:
ANTIPHOSPHOLIPID ANTIBODY SYNDROME (APS) ○ Propagation
● APS is an autoimmune disorder characterized by: ○ Embolization: Thrombi dislodge and travel to other sites.
○ Presence of one or more antiphospholipid (aPL) ○ Dissolution by fibrinolytic activity
autoantibodies ○ Organization and recanalization: Ingrowth of Endothelial cells,
○ Venous or arterial thromboses, or pregnancy complications such smooth muscle cells, and fibroblasts to create vascular channels
as recurrent miscarriages, unexplained fetal death, and premature or incorporate the thrombus into the vessel wall.
birth. ● Rarely, microbial seeding of a thrombus leads to a mycotic
● APS may be primary or secondary. aneurysm.
○ Primary antiphospholipid syndrome. VENOUS THROMBOSIS (PHLEBOTHROMBOSIS)
■ Patients exhibit only the manifestations of a hypercoagulable
● Superficial thrombus usually occur in varicose saphenous veins,
state and lack evidence of other well-defined autoimmune
causing local congestion and pain but rarely embolizing. Local
disorders.
edema and imparied venous drainage predispose to skin infections
○ Secondary antiphospholipid syndrome
and varicose ulcers
■ Seen in individuals with a well-defined autoimmune disease,
● Deep thrombus in larger leg veins above the knee (e.g., popliteal,
such as systemic lupus erythematosus.
femoral, and iliac veins) can result in pain and edema, as well as
■ Also termed as lupus anticoagulant syndrome.
increased risk for embolization. Venous obstruction is usually offset
● Approximately 50% of the patients with APS have the primary form,
by collateral flow, and deep vein thromboses are asymptomatic in
and the rest occur in association with well-defined autoimmune
approx. 50% of patients, being recognized only after embolization
disease, most commonly SLE.
● DVT occurs in multiple clinical settings:
CLINICAL PRESENTATIONS OF APS: ○ Advanced age, bed rest, or immobilization, diminishing the milking
● The clinical manifestations of APS are varied: action of muscles in the lower leg and slowing venous return
○ They include recurrent thromboses, repeated miscarriages, ○ Congestive heart failure(CHF)
cardiac valve vegetations, and thrombocytopenia. ○ Trauma, surgery, and burns result in reduced physical activity,
● Pulmonary embolism (following lower extremity venous injury to vessels, release of procoagulant substances from
thrombosis), tissues, and reduced t-PA
● Pulmonary hypertension (from recurrent subclinical pulmonary ○ The puerperal and postpartum states are associated with amniotic
emboli), fluid embolization and hypercoagulability
● Valvular heart disease, stroke, bowel infarction, or renovascular ○ Tumor-associated procoagulant release (migratory
hypertension. thrombophlebitis or Trousseau syndrome)
● The aPL antibodies are directed against anionic membrane ARTERIAL AND CARDIAC THROMBOSIS
phospholipids or proteins associated with phospholipids.
● Atherosclerosis is the major cause of arterial thrombi which is due
● Proteins that are recognized by these antibodies include cardiolipin
to abnormal flow and endothelial damage.
and β2-glycoprotein I.
● Myocardial infarction with dyskinesis and endocardial damage can
○ This glycoprotein is found in plasma, but it has strong avidity for
cause mural thrombi.
phospholipids expressed on the surfaces of endothelial cells,
● Rheumatic valvular disease resulting in mitral valve scarring and
monocytes, platelets, thrombin, and trophoblasts.
stenosis with left atrial dilation predisposes to atrial thrombus
● Anti–β2-glycoprotein antibodies are suspected to have a major role
formation
in APS by activating endothelial cells, monocytes, and platelets.
● Atrial fibrillation augments the blood stasis and propensity to
● Patients with APS also show evidence of complement activation and
thrombose.
inhibition of fibrinolytic processes, both of which favor the
● Cardiac and aortic mural thrombi can embolize peripherally: brain,
prothrombotic state.
kidneys, and spleen are prime targets.
● Pregnancy morbidity is a defining manifestation of APS
○ Fetal loss - not due to thrombosis, but from antibody-mediated
interference with the growth and differentiation of trophoblasts,
leading to a failure of placentation.
MORPHOLOGY
● Venous thrombi characteristically occur in sites of stasis are
occlusive
● Arterial or cardiac thrombi usually begin at sites of endothelial injury.
○ Aortic or cardiac thrombi are typically nonocclusive due to rapid
and high-volume flow in those sites. FIGURE 22. Mural Thrombi (A) Thrombus in the left and right ventricular apices
○ Smaller arterial thrombi can be occlusive. (arrows), overlying a white fibrous scar (B) Laminated thrombus in a dilated
● Thrombi are generally firmly attached at their site of origin and abdominal aortic aneurysm (asterisks). Numerous friable mural thrombi are also
typically propagate toward the heart. superimposed on advanced atherosclerotic lesions of the more proximal aorta
(left side of picture)
● Arterial and cardiac mural thrombi have gross and microscopic
laminations produced by pale layers of platelets and fibrin D. DISSEMINATED INTRAVASCULAR COAGULATION
alternating with darker erythrocyte-rich layers.
● Venous thrombi typically occur in a relatively static environment,
(DIC)
resulting in a fairly uniform cast containing abundant erythrocytes ● Reflected by widespread fibrin microthrombi in the microcirculation
among sparse fibrin strands. ● Caused by disorders ranging from obstetric complications to
● Phlebothrombosis most commonly affects the veins or lower advanced malignancy
extremities. ● Not a primary disease but rather a complication of any diffuse
● Valve thrombosis thrombin activation
○ Infective endocarditis: organism form large infected thrombotic ● Microthrombi can cause diffuse circulatory insufficiency, particularly
mases, with associated valve damage and systemic infection in the brain, lungs, heart, and kidneys
○ Nonbacterial thrombotic endocarditis: non infected, sterile ● There is also concurrent consumption of platelets and coagulation
vegetations develop in hypercoagulable states, typically without factors (consumption coagulopathy) with fibrinolytic pathway
valve damage. activation, leading to uncontrollable bleeding
○ Verrucous (Libman-Sacks) endocarditis (sterile vegetations)
occurs in systemic lupus erythematosus due to immune complex
deposition; inflammation can cause valve scarring

7
B. SYSTEMIC THROMBOEMBOLISM
● Most systemic emboli (80%) arise from intracardiac mural thrombi,
⅔ of which are associated with left ventricular wall infarcts and
another ¼ with left atrial dilation and fibrillation
● Remainder originates from aortic aneurysms, atherosclerotic
plaques, valvular vegetations, or venous thrombi (paradoxical
emboli); 10 to 15% are of unknown origin
● In contrast to venous emboli, the vast majority of which lodge in the
lung, arterial emboli can travel to a wide variety of sites; the point of
arrest depends on the source and the relative amount of blood flow
that downstream tissues receive
● Most come to rest in the lower extremities (75%) or the brain (10%),
but other tissues, other including the intestines, kidneys, spleen, and
upper extremities, may be involved
● The consequences of systemic emboli depend on the vulnerability of
the affected tissues to ischemia, the caliber of the occluded vessel,
and whether a collateral blood supply exists; in general, however,
the outcome is tissue infarction

C. FAT EMBOLISM
● Microscopic fat globules may be found in the circulation after
FIGURE 23. Thrombosis fractures of long bones (which have fatty marrow) or, rarely, in soft
tissue trauma and burns.
IV. EMBOLISM ● Fat is released by marrow or adipose tissue injury and enters the
● Refers to any intravascular solid, liquid, or gaseous mass carried by circulation through rupture of the blood vessels.
blood flow to a site distant from its origin. ● Less than 10% of patients with fat embolism have any clinical
● Most (99%) arise from thrombi, hence the term thromboembolism findings.
● Rare forms include fat droplets, gas bubbles, atherosclerotic debris ● Fat embolism syndrome is characterized by pulmonary
(atheroemboli), tumor fragments, bone marrow, or foreign bodies insufficiency, neurologic symptoms, anemia, and thrombocytopenia.
(e.g., bullets).
● Emboli lodge in vessels too small to permit further passage,
resulting in partial or complete vascular occlusion and ischemic
necrosis (infarction).
THROMBOEMBOLISM
● Embolism causing blockage is derived from a thrombus
○ Pulmonary Thromboembolism
■ Thrombus (usually from a DVT) breaks off and goes to the
right ventricle. From there it is pumped out to the lungs and
blocks pulmonary arteries. The problem at first is not the
ischemia per se, but instead that this blood is not oxygenated
and does not return to the heart (thus eventually causing
systemic ischemia). Figure 24. Fat Embolism
○ Systemic Thromboembolism
■ Thrombus originates in the left ventricular wall or wall of the D. AIR EMBOLISM
aorta breaks off and causes infarction at a distant site (brain, ● Gas bubbles within the circulation can obstruct vascular flow (and
kidney, spleen). cause distal ischemic injury) acting as thrombotic masses. Bubbles
may coalesce to form frothy masses sufficiently large to occlude
A. PULMONARY EMBOLISM (PE) major vessels.
● Pulmonary emboli (PE) occur in 0.2% to 0.4% of hospitalized ● Air may enter the circulation during obstetric procedures or as a
patients and cause approximately 100,000 deaths annually in the consequence of chest wall injury.
United States. ● An excess of 100 cc is required to have a clinical effect.
● Greater than 95% of PE originate from DVT, although DVT are 2 to 3
times more common than PE.
● Can occlude the main pulmonary artery, impact across the
bifurcation (saddle embolus), or pass into smaller arterioles.
● Multiple emboli can occur, either sequentially or as a shower of small
emboli from a single large mass; in general, one PE puts a patient at
risk for more.
● Rarely emboli pass through atrial or ventricular defects into the
systemic circulation (paradoxical embolism).
● Most PE (60% to 80%) are small and clinically silent. They
eventually organize and get incorporated into the vessel wall or
leave a delicate, bridging fibrous web.
● Sudden death, right-sided heart failure (cor pulmonale), or
cardiovascular collapse occurs when 60% or more of the pulmonary
circulation is obstructed with emboli
● PE in medium-sized arteries can cause pulmonary hemorrhage but
usually not pulmonary infarction due to collateral bronchial artery
flow. However, with left-sided cardiac failure (and diminished Figure 25. Air Embolism in Chest X-ray
bronchial circulation), infarcts can result
● PE in small end-arteriolar vessels will typically cause hemorrhage or
infarction
● Multiple emboli over time can cause pulmonary hypertension and
right ventricular failure

8
MORPHOLOGY
Infarcts are classified:
○ On the basis of their color (reflecting the amount of
hemorrhage)
■ Either red (hemorrhagic) or white (anemic)
○ Presence (septic infarct) or absence of microbial infection
(bland infarct).

RED INFARCTS
● Red (hemorrhagic) infarcts occur:
○ With venous occlusions (such as in ovarian torsion)
○ In loose tissues (such as lung), and in tissues with dual
circulations (e.g., lung and small intestine), permitting flow of
blood from the unobstructed vessel into the affected zone
Figure 26. Air Embolism

E. AMNIOTIC FLUID EMBOLISM


● A grave and uncommon complication of labor and the immediate
postpartum period, characterized by sudden severe dyspnea,
cyanosis, and hypotensive shock, followed by seizures and coma.
● If the patient survives the initial crisis, pulmonary edema develops,
along with DIC, owing to release of thrombogenic substances from
amniotic fluid.

Figure 28. Recent Hemorrhagic Infarct

WHITE INFARCTS
● White (anemic) infarcts occur:
○ With arterial occlusions in solid organs with end-arterial circulation
■ Heart, spleen, and kidney
Figure 27. Epithelial Squames in a Peripheral Pulmonary Artery
■ Where the solidity of the tissue limits the amount of
● Amniotic fluid may gain access to uterine veins following a tear in hemorrhage that can seep into the area of ischemic necrosis
the placental membranes and embolize to the lungs, producing from adjoining capillary beds.
acute dyspnea with cyanosis and shock.
● Fetal squames, lanugo hair, vernix, and mucin can embolize small
pulmonary arteries.
● Caused by infusion of amniotic fluid or fetal tissue into the maternal
circulation via a tear in the placental membranes or rupture of
uterine veins.
● Microscopy: presence in the pulmonary microcirculation of
squamous cells shed from fetal skin, lanugo hair, fat from vernix
caseosa, and mucin derived from the fetal respiratory or
gastrointestinal tract. Marked pulmonary edema and diffuse alveolar
damage are also present. Systemic fibrin thrombi indicative of DIC
can also be seen.

V. INFARCTION
● Area of ischemic necrosis caused by occlusion of arterial supply or
venous drainage. The nature and extent of damage is influenced by:
Figure 29. Anemic Infarct
○ Nature of blood supply: tissues with dual or collateral blood
supply (e.g. lungs, liver, and limbs) are less affected compared to
MORPHOLOGY
end organs (muscles, brain, kidney, spleen).
○ Rate of development: slowly progressing occlusion tolerated GROSS:
because of development of collateral routes ○ Wedge-shaped
○ Tissue vulnerability: neurons can withstand only 3-4 minutes of ○ Occluded vessel at the apex
hypoxia, myocytes ~30 minutes, fibroblasts can survive many ○ Periphery of the organ forming the base.
hours in low oxygen.
MICRO:
○ Inflammatory response begins
■ Along the margins of infarcts
■ Within a few hours
■ Well defined (within 1 or 2 days), followed by
○ Gradual degradation of the dead tissue with phagocytosis
(neutrophils and macrophages).
○ Infarcts are replaced by scar tissue.

9
VI. SHOCK chemotactic fragments (C5a), and opsonins (C3b), all of which
● Shock is a state of circulatory failure that impairs tissue perfusion contribute to the pro-inflammatory state.
and leads to cellular hypoxia. ○ Microbial components can activate coagulation directly through
● At the outset, the cellular injury is reversible; however, prolonged factor XII and indirectly through altered endothelial function
shock eventually leads to irreversible tissue injury and can be fatal. ○ The accompanying widespread activation of thrombin may further
● Shock may complicate severe hemorrhage, extensive trauma or augment inflammation by triggering protease-activated receptors
burns, myocardial infarction, pulmonary embolism, and microbial on inflammatory cells.
sepsis. ○ The hyperinflammatory state, initiated by sepsis, triggers
● Types of shock counter-regulatory immunosuppressive mechanisms, which may
○ Cardiogenic shock involve both innate and adaptive immune cells.
■ Results from myocardial pump failure ○ Proposed mechanisms for the immune suppression include a shift
■ E.g intrinsic myocardial infarction, ventricular arrhythmias. from proinflammatory (Th1) to anti-inflammatory (Th2) cytokines,
○ Hypovolemic shock production of anti-inflammatory mediators (e.g., soluble TNF
■ Results from loss of blood or plasma volume receptor, IL-1 receptor antagonist, and IL-10), lymphocyte
■ E.g. hemorrhage, fluid loss from severe burns, or trauma. apoptosis, the immunosuppressive effects of apoptotic cells, and
○ Septic shock the induction of cellular anergy.
■ Caused by systemic microbial infection. ● Endothelial activation and injury
■ Most commonly due to gram negative infections (endotoxic ○ The pro-inflammatory state and endothelial cell activation
shock), but it can also occur with gram-positive and fungal associated with sepsis lead to widespread vascular leakage and
infections. tissue edema, which have deleterious effects on both nutrient
● Cytokine storm of TNF, IL-1, and IL-6 in response to bacterial delivery and waste removal.
antigens (usu. lipopolysaccharides from gram-negative bacilli) ○ One effect of inflammatory cytokines is to loosen endothelial cell
resulting in tight junctions, making vessels leaky and resulting in the
○ Systemic vasodilation accumulation of protein-rich edema fluid throughout the body.
○ Reduced cardiac contractility ○ Activated endothelium also upregulates production of NO and
○ Widespread endothelial injury and activation other vasoactive inflammatory mediators (e.g., C3a, C5a, and
○ Systemic activation of clotting cascade (Disseminated PAF), which may contribute to vascular smooth muscle relaxation
Intravascular Coagulation or “DIC”) and systemic hypotension.
○ Another feature of sepsis is microvascular dysfunction.
A. PATHOGENESIS OF SEPTIC SHOCK ■ There is an increase in capillaries with intermittent flow, and
● The pathogenesis of sepsis is a combination of direct microbial heterogeneity of flow in various capillary beds, and the normal
injury and activation of host inflammatory responses autoregulation of flow based on tissue metabolic environment is
● Most cases of septic shock are now caused by gram-positive lost.
bacteria, followed by gram-negative bacteria and fungi ■ These changes cause a mismatch in oxygen needs and
● Superantigens oxygen delivery.
○ Class of secreted bacterial products ● Induction of a procoagulant state
○ Can also cause a similar syndrome (e.g, toxic shock syndrome) ○ The derangement in coagulation is sufficient to produce DIC in up
by inducing polyclonal T-cell activation and the systemic release to one-half of septic patients.
of high levels of proinflammatory cytokines ○ Pro-inflammatory cytokines increase tissue factor production by
● Morbidity and mortality in sepsis are consequences of tissue monocytes and possibly endothelial cells as well, and decrease
hypoperfusion and multiorgan dysfunction despite initially preserved the production of endothelial anticoagulant factors, such as tissue
or even increased cardiac output. factor pathway inhibitor, thrombomodulin, and protein C
○ This is due to systemic vasodilation accompanied by widespread ○ Some studies suggest a role for neutrophil extracellular traps
EC activation and injury, leading to a hypercoagulable state and (NETs) in promoting the procoagulant state by stimulating both
DIC. intrinsic and extrinsic pathways of coagulation.
○ There are also systemic metabolic changes that suppress normal ○ The vascular leak and tissue edema decrease blood flow at the
cellular function level of small vessels, producing stasis and diminishing the
● A similar widespread inflammatory response - systemic washout of activated coagulation factors.
inflammatory response syndrome - can be triggered after ○ These effects lead to systemic activation of thrombin and the
extensive trauma or burn injury, in the absence of any infection deposition of fibrin-rich thrombi in small vessels, often throughout
● The severity and outcome of septic shock are likely dependent on: the body, further compromising tissue perfusion.
○ the extent and virulence of the infection ○ In full-blown DIC, the consumption of coagulation factors and
○ the immune status of the host platelets is so great that deficiencies of these factors appear,
○ the presence of other comorbid conditions leading to concomitant bleeding and hemorrhage
○ the pattern and level of mediator production. ● Metabolic abnormalities
○ Septic patients exhibit insulin resistance and hyperglycemia.
FACTORS BELIEVED TO PLAY MAJOR ROLES IN THE ○ Cytokines such as TNF and IL-1, stress-induced hormones (such
PATHOPHYSIOLOGY OF SEPTIC SHOCK: as glucagon, growth hormone, and glucocorticoids), and
● Inflammatory and counter-inflammatory responses. catecholamines all drive gluconeogenesis.
○ In sepsis, various microbial cell wall constituents engage ○ The pro-inflammatory cytokines suppress insulin release while
receptors on cells of the innate immune system, triggering simultaneously promoting insulin resistance in the liver and other
proinflammatory responses. tissues, likely by impairing the surface expression of glucose
■ Ligation of these receptors leads to increased expression of the transporter-4 (GLUT-4).
genes encoding inflammatory mediators via activation and ○ Hyperglycemia decreases neutrophil function—thereby
nuclear translocation of the transcription factor nuclear suppressing bactericidal activity—and causes increased adhesion
factor-κB (NF-κB). molecule expression on endothelial cells
○ Markers of acute inflammation such as C-reactive protein and ○ Although sepsis is initially associated with an acute surge in
procalcitonin are also elevated. glucocorticoid production, this phase may be followed by adrenal
○ Reactive oxygen species and lipid mediators such as insufficiency and a functional deficit of glucocorticoids
prostaglandins and platelet-activating factor (PAF) are also ○ Cellular hypoxia and diminished oxidative phosphorylation lead to
elaborated. increased lactate production and lactic acidosis.
■ These effector molecules induce endothelial cells (and other ● Organ dysfunction
cell types) to upregulate adhesion molecule expression and ○ Systemic hypotension, interstitial edema, microvascular
further stimulate cytokine and chemokine production. dysfunction, and small vessel thrombosis all decrease the delivery
○ The complement cascade is also activated by microbial of oxygen and nutrients to the tissues that, because of cellular
components, both directly and through the proteolytic activity of hypoxia, fail to properly use those nutrients that are delivered.
plasmin, resulting in the production of anaphylatoxins (C3a, C5a), ○ Mitochondrial damage resulting from oxidative stress impairs
oxygen use

10
○ High levels of cytokines and secondary mediators diminish
myocardial contractility and cardiac output
○ Increased vascular permeability and endothelial injury can lead to
the acute respiratory distress syndrome
○ These factors may conspire to cause failure of multiple organs,
particularly the kidneys, liver, lungs, and heart, culminating in
death.

B. STAGES OF SHOCK
● Nonprogressive stage
○ Reflex compensatory mechanisms (tachycardia, peripheral
vasoconstriction, renal fluid retention) compensate for
hypoperfusion.
○ Causes common symptoms associated with shock:
■ Weak, rapid pulse
■ Shallow, rapid breathing
■ Cool, clammy skin (the exception is septic shock that may
present with flushing due to widespread inflammatory
response).
● Progressive stage
○ Characterized by tissue hypoperfusion and onset of worsening
circulatory and metabolic derangement, including acidosis
○ Metabolic lactic acidosis blunts vasomotor response and blood
starts to pool in peripheral tissues (increasing hypercoagulative
risk), vital organs perfused less and begin to fail.
○ Clinical symptoms associated with this phase are reduced urine
output, acidosis, and electrolyte imbalances
● Irreversible stage
○ Widespread tissue necrosis induces systemic inflammatory
response (vasodilation, etc.), reduced cardiac function, and acute
renal failure.
○ Cellular and tissue injury is so severe that even if the
hemodynamic defects are corrected, survival is not possible

C. CLINICAL COURSE
● In hypovolemic and cardiogenic shock, patients exhibit hypotension,
a weak rapid pulse, tachypnea, and cool, clammy, cyanotic skin
● In septic shock, the skin may be warm and flushed owing to
peripheral vasodilation.
● The primary threat to life is the underlying initiating event (e.g.,
myocardial infarction, severe hemorrhage, bacterial infection)
● As shock progresses, electrolyte disturbances and metabolic
acidosis (lactic acidosis) complicate the situation followed by
progressive fall in urine output.
● Prognosis varies with the origin of shock and its duration.
○ More than 90% of young, otherwise healthy patients with
hypovolemic shock survive with appropriate management
○ Comparis septic or cardiogenic shock is associated with
substantially poorer outcomes, even with state-of-the-art care.

MORPHOLOGY
● The cellular and tissue changes induced by shock are
essentially those of hypoxic injury, since shock is characterized
by failure of multiple organ systems, the cellular changes may
appear in any tissue.
● They are particularly evident in brain, heart, lungs, kidneys,
adrenals, and gastrointestinal tract
● Brain - ischemic encephalopathy
● Heart - coagulation necrosis, may exhibit subendocardial
hemorrhage and/or contraction band necrosis.
● Kidneys - tubular ischemic injury (acute tubular necrosis,
therefore oliguria, anuria, and electrolyte disturbances constitute
major clinical problems.
● Lungs - seldom affected in pure hypovolemic shock because
they are resistant to hypoxic injury. When shock is caused by
bacterial sepsis or trauma, however, changes of diffuse alveolar
damage may appear, the so called shock lung

REFERENCES
● Kumar, V., Abbas, A. K., Aster, J. C., & Perkins, J. A. (2018). Robbins Basic
Pathology (Tenth edition.). Philadelphia: Elsevier.
● Dr. Nicolas, R. (2022). Hemodynamic Disorders, Thromboembolic Disease,
and Shock.

11
APPENDIX

APPENDIX A. Sequence of Events Leading to Hemostasis at a Site of Vascular Injury

APPENDIX B. Anticoagulant activities of normal endothelium

12
APPENDIX C. Mechanism of APPENDIX D. Thrombus (microscopic)
Heparin-induced Thrombocytopenia

APPENDIX E. Antemortem Thrombi vs. Postmortem Clump

13
LE 1.01 THE CELL AS A UNIT OF HEALTH AND DISEASE
September 05, 2022
● Introns
OUTLINE ○ Segment of a DNA or RNA that does not code for proteins and
I. Introduction G. Golgi Apparatus usually gets spliced off.
II. Organization of Nuclear DNA H. Endosomes ● Exons
III. Human Genome I. Lysosomes ○ A coding region of a gene that contains information necessary to
A. Single Nucleotide J. Peroxisomes encode a protein.
Polymorphisms (SNPs) K. Proteasomes
B. Copy Number Variations L. Cytoskeleton III. HUMAN GENOME
(CNVs) VI. Cellular Communication ● The human genome is made up of roughly 3.2 billion DNA base
IV. Epigenetics A. Cell SIgnaling
pairs (within the genome there are only 20,000 protein-encoding
A. Epigenetic Mechanism B. Signal Transduction
B. MicroRNA & Long Non C. Transcription Factors
genes)
Coding RNA D. Growth Factors and ● 1.5% of genes are devoted to making proteins (protein coding
V. Cellular Organelles Receptors genes)
A. The Cell E. Extracellular Matrix (ECM) ● The remaining are all non-coding genes that accounts for 98.5%
B. Cell Membrane VII. Cell Cycle ● There are 5 major classes of functional non-protein coding
C. Cytoplasm A. Check Point sequences in the human genome:
D. MItochondria B. Cyclins & Cyclin-Dependent ○ Promoter and enhancer regions
E. Ribosome Kinases ○ Binding sites for factors that organize the chromatin structure
F. Endoplasmic Reticulum VIII. Stem Cell ○ Non-coding regulatory RNAs
■ Micro-RNAs (miRNAs)
LEGEND ■ Long noncoding RNAs (lncRNAs)
Remember Lecturer Book Presentation ○ Mobile genetic element (e.g., transposons)
○ Telomeres (chromosome ends) and centromeres (chromosome
“tethers”)
I. INTRODUCTION ● At a genomic level, any two individuals share more than 99.5% of
● Pathology literally translates as the study of suffering (Greek pathos their DNA sequences. The less than 0.5% which corresponds to
= suffering, logos = study) roughly 50M base pairs is the interest of pathology.
○ Applied to modern medicine, it is the study of disease ● Many genetic variations (polymorphisms) associated with diseases
● The foundation of modern pathology is understanding the cellular are located in the non-protein coding regions of the genome
and molecular aberrations that give rise to diseases ● The 2 most common form of DNA variation in the human genome:
● Rudolf Virchow ○ Single Nucleotide Polymorphisms (SNPs)
○ Father of modern pathology ○ Copy Number Variations (CNVs)
○ States that disease originates at the cellular level; cellular
pathologies arise from perturbations in molecules (genes,
proteins, and metabolites) that influence cell survival and
behaviors

II. ORGANIZATION OF NUCLEAR DNA

FIGURE 2. Human Genome

A. SINGLE NUCLEOTIDE POLYMORPHISMS (SNPs)

FIGURE 1. The Organization of Nuclear DNA

● Euchromatin
○ The genetic material is organized into dispersed and
transcriptionally active form.
● Heterochromatin
○ The genetic material is organized into dense and transcriptionally
inactive form.
● Telomeres
○ Repetitive nucleotide sequences that cap the termini of
chromatids and permit repeated replication without deterioration FIGURE 3. Substitution of a Single Nucleotide Occurs at a Specific Location in
of genes near the ends. the Genome
● Promoters
● The most common genetic variations among human beings and
○ Noncoding regions of DNA that initiate gene transcription; they
represents a difference in a single nucleotide.
are on the same strand and upstream of their associated gene.
● Are SNPs a mutation?
● Histones
○ SNP are a type of mutation
○ Highly dynamic octamer core proteins where the DNA strands are
○ But not all mutations are SNPs
compressed/wrapped forming a nucleosome.
● What is the difference between SNPs and Mutations?
● Centromeres
○ SNPs – the frequency of occurrence is more that 1% of the
○ Links a pair of sister chromatids
population

1
○ Mutation – The frequency of occurrence is less than 1% of the A. EPIGENETIC MECHANISM
population
○ SNPs are harmless; mutations are potentially harmful.
● Why do mutations occur in less than 1%?
○ Mutations happen randomly in different locations across different
individuals
● Some SNPs, termed “neutral” variants, are thought to have no effect
on gene function or individual phenotype

B. COPY NUMBER VARIATIONS (CNVs)


Note: See Appendix A
● Is a type of structural/genetic variation consisting of different
numbers of large contiguous stretches of DNA
● CNVs are responsible for 5 million to 24 million base pairs of
sequence difference between any two individuals
● Most of these variations are harmless and contribute to the process FIGURE 5. An Electron Micrograph of a Nucleus
of evolution.
● On the other hand, if it removes an important gene or significant ● For DNA to fit inside a nucleus it has to exist as a condensed
chunk of a genetic material then health problems ensue. chromatin form.
● 50% of CNVs involve gene-coding sequences ● Heterochromatin: histochemically dense and transcriptionally
○ May underlie a large portion of human phenotypic diversity inactive (Blackened/darker area)
● Alterations in DNA sequence cannot by themselves explain the ● Euchromatin: histochemically dispersed and transcriptionally active
diversity of phenotypes (Lighter area)
○ Classic genetic inheritance cannot explain differing phenotypes in ● Ways of altering the cells factory settings without changing the
monozygotic twins factory blueprint.
○ The answer to these conundrums lies in epigenetics which are the ○ Histones
heritable changes in gene expression that are not caused by ○ DNA Methylation – suppress gene expression.
variations in DNA sequence ○ Post-translational Histone Modifications
● Dosage sensitivity – reflects the general relationship between the ○ Non-coding RNAs:
gene copy number and the protein product. ■ MicroRNAs(miRNAs)
■ LongRNAs(lncRNAs)
IV. EPIGENETICS ○ E.g., XIST (X-inactive specific transcript)
● Epigenetics is taking the genes that we have and manipulating their ● Histone Methylation
expressions without changing the DNA sequence itself. ○ Both Lysine and Arginine can be methylated
● These changes can be passed on from one generation to another ○ Leads to transcriptional activation or repression depending on
● Epigenetics is a stable and heritable process which histone residue.
○ Your genetic blueprints do not purely dictate your biological fate. ● Histone Acetylation
There are factors outside your DNA sequence that can ○ Lysine residues acetylated by histone acetyltransferases (HATs)
significantly affect you as an organism. that opens chromatin for transcription
○ Histone deacetylases (HDACs) reverses the acetylation resulting
to chromatin condensation
● Histone Phosphorylation
○ Serine residues modified by phosphorylation
○ Depending on the specific residue, the DNA may be opened for
transcription or condensed and inactive.
○ DNA Methylation
○ High levels of DNA methylation in gene regulatory elements
typically result in chromatin condensation and transcriptional
silencing
○ Tightly regulated by methyltransferases, demethylating enzymes,
and methylated-DNA-binding-proteins
○ DNA methylation is adding a methyl group to a carbon number 5
which eventually turns off the gene (transcriptional silencing)
○ Methylation mutes DNA
● Non-coding RNAs are encoded by genes that are transcribed but
FIGURE 4. Agouti Mice
not translated
● Nucleosomes are comprised of octamers of histone proteins (two
● Back in 2000, Randy Jirtle and Robert Waterland designed a each of histone subunits H2A, H2B, H3, and H4)
groundbreaking genetic experiment to see if they could change the ● The relative state of DNA unwinding regulated by histone
genetic legacy of Agouti mice. modification, including acetylation, methylation, and/or
● Fat Yellow mice known to scientists as agouti mice, so called phosphorylation
because they carry a particular gene – the Agouti gene-making them
ravenous and yellow and renders them prone to cancer and
diabetes
● When they breed, most offspring are identical: yellow and fat and
susceptible to the life-shortening disease.
● Parent mice, however, produced a majority of offspring that looked
different altogether – slender, mousy brown, and no susceptibility to
cancer and diabetes. They lived to a spry old age. Modifying the diet
of the parent mice (increased methyl groups) affected the gene and
silenced the DNA causing obesity and diabetes thru DNA
methylation.
● Thus, the effect of the agouti gene had been erased.
● Your DNA is not your destiny.
Note: See Appendix B FIGURE 6. DNA Methylation – If you add a methyl group you are turning the
DNA off

2
● “Calico Cat Genetics”
● Its black and orange patches define the Calico cat
● The pigmentation of this cat is X-linked. Which means the blueprint
for this animal’s color is on the X chromosomes.
○ Females have double X chromosomes resulting in a potentially
toxic double dose of X-linked genes (an X chromosome contains
1,000 genes for proper development and viability)
○ Dosage Compensation genetic mechanism solves the imbalance
of extra copies of X chromosomes via X chromosome inactivation
● In humans, females shut down one of their X chromosomes during
early embryogenesis through “X Chromosome Inactivation”.
○ Inactivated through a process that slathers the chromosome with
small RNA
○ RNA molecules initiate other changes that lead to the X
chromosome being condensed into a small, dense cluster of
FIGURE 7. Histone: Acetylation and Methylation DNAs called the “Barr Body”
● This blueprint has two versions or alleles.
○ One version code for an orange (XB – orange)
B. MicroRNA & LONG NON CODING RNA
○ The other version code for black (Xb – black)
● Genes can also be regulated by noncoding RNAs in which these
● If the orange is dominant and black is recessive, the question is if
genomic sequences are transcribed but not translated. 2 examples
cats inherit a blueprint for orange, shouldn’t the animal be purely
are the micro-RNAs and long noncoding RNAs.
orange?
MicroRNA (miRNAs) ● Why do they get patches of orange and black?
○ This is caused by X-linked inactivation
● Short RNAs (average of 22 nucleotides) function primarily to ● What explains the white fur?
modulate the translation of target mRNAs into their corresponding ○ Epistasis – describes how genes interact and how it can affect
proteins. other phenotypes. It means the genes can mask each other’s
● miRNAs do not encode proteins; they modulate translation of target presence to produce an entirely new trait.
messenger RNAs (mRNAs) ● One of the two chromosomes in cat skin cells gets switched off by
● miRNA is a type of small RNA that can bind to a normal messenger supercoiling into a structure called a Barr body – irreversible process
RNA molecule and prevent mRNAs from leaking proteins lyonization
○ Subsequent base pairing between the miRNA strand and its ● Lyonization is named after the British geneticist Mary Lyon.
target mRNA directs the RISC (RNA-induced silencing complex) ● Due to its lyonization process only the uncoiled version of the
to either induce mRNA cleavage or repress its translation (post blueprint is expressed in each skin cell.
transcriptionally silenced) ● If you get a skin sample from the orange area and you get cells with
● The miRNA genes make the primary miRNA (pri-miRNA) genes an active XB and an inactive Xb and vice versa if you get samples
which are processed further into pre-miRNA. from the black area – it is called a mosaic
● The pre-miRNA leaves the nucleus. ● Mosaic – when you have two or more genetically different cell lines
● The DICER enzyme chops up the pre-miRNA to yield a mature derived from a single zygote. Comes from a single fertilized egg.
double stranded miRNA. ● Chimera – genetically defined as cells with independent origin and
● The double stranded miRNA unwinds to a single strand. genotype; contains at least two different sets of DNA. Comes from
● The strand joins the RNA induced silencing complex (RISC). two fertilized eggs.
● The pairing of miRNA and tRNA causes this complex to either ● Two types of X chromosome inactivation
cleave the mRNA or suppress its translation. 1. Imprinted – the maternal/paternal X chromosome is preferentially
● Both of these outcomes lead to gene silencing. silenced in the placenta of mammals.
2. Random – occurs in early female embryos where both the
LONG NONCODING RNA (lncRNAs) maternal and paternal X chromosomes have an equal chance of
● Modulate gene expression in many ways becoming inactivated.
○ Bind to regions of chromatin, restricting RNA polymerase access
GENE EDITING
to coding genes within the region.
● Best known example is XIST (X-inactive Specific Transcript) ● New development that allows high-fidelity genome editing may
○ Transcribed from X chromosome and plays an essential role in usher in the next era of the molecular revolution.
physiologic X chromosome inactivation ● Clustered Regularly Interspaced Short Palindromic Repeats
○ XIST is an RNA gene on the X chromosome of a placental (CRISPRs) & CRISPR-associated genes (Cas), such as the Cas9
mammal that acts as a major effector of X inactivation process nuclease
○ This type of RNA is transcribed by X chromosome that binds and ○ Linked genetic elements that endow prokaryotes with a form of
represses X expression to prevent access of RNA polymerase acquired immunity to phages and plasmids
○ Responsible for random X inactivation in females ○ CRISPR segments are subsequently transcribed and processed
○ XIST itself escapes X inactivation but forms a repressive cloak on into guide RNA sequences that it can be cleaved to disable the
the X chromosome from which it is being transcribed, resulting in infecting agent
gene silencing ○ Gene editing repurposes this process by using
○ artificial 20-base guide RNAs (gRNAs) that bind Cas9 and are
complementary to a targeted DNA sequence
○ Applications include inserting specific mutations in cells and
tissues to model cancers and other diseases and rapidly
generating transgenic animal models from edited embryonic stem
cells
○ Makes it possible to selectively edit mutations that cause heritable
diseases and probably eliminate less desirable traits
● Process Of Gene Editing
1. In bacteria, DNA sequences consisting of CRISPRs are
transcribed into guide RNAs (gRNAs) with a constant region and
a variable sequence of about 20 bases.
2. The constant regions of gRNAs bind to Cas9, permitting the
variable regions to form heteroduplexes with homologous host
cell DNA sequences.
Figure 8. Calico cat: Female

3
3. The Cas9 nuclease then cleaves the bound DNA, producing a ● 6 Categories of Integral Proteins based on Functions:
double-stranded DNA break. 1. Pumps
4. To perform gene editing, gRNAs are designed with variable 2. Channels
regions that are homologous to a target DNA sequence of 3. Receptors
interest. Coexpression of the gRNA and Cas9 in cells leads to 4. Linkers
efficient cleavage of the target sequence. In the absence of 5. Enzymes
homologous DNA, the broken DNA is repaired by nonhomologous 6. Structural proteins
end joining (NHEJ), an error-prone method that often introduces ● Passive Transport – moves molecules from higher concentration to
disruptive insertions or deletions (indels). lower concentration.
5. By contrast, in the presence of a homologous “donor” DNA ○ Forms of passive transport:
spanning the region targeted by CRISPR/Cas9, cells instead may ■ Diffusion – Ex. O2, CO2, Steroid based estradiol, Vitamin D
use homologous DNA recombination (HDR) to repair the DNA ■ Facilitated Diffusion – Ex. Water, ethanol, urea
break. HDR is less efficient than NHEJ, but has the capacity to ■ Hypertonicity – extracellular salt in excess of that in the
introduce precise changes in DNA sequence. cytoplasm; causes net movement of water out of the cells
6. Potential applications of CRISPR/Cas9 coupled with HDR include ■ Hypotonicity – causes net movement of water into cells
the repair of inherited genetic defects and the creation of ● Active Transport – moves molecules from lower concentration to
pathogenic mutations. higher concentration.
○ E.g., Transferrin and LDL
V. CELLULAR ORGANELLES ○ Multidrug resistance (MDR) protein – pumps polar compounds
(e.g., chemotherapeutic drugs) out of cells; may render cancer
A. THE CELL cells resistant to treatment
● Basic functional unit of all multicellular organisms ● Carriers and Channels – required for uptake and secretion of ions
● Cell housekeeping functions to maintain intracellular homeostasis: and larger molecules that are required for cellular function
○ Protection from environment ○ Channel proteins – create hydrophilic pores which when open,
○ Nutrient acquisition permit rapid movement of solutes
○ Metabolism ○ Carrier proteins – bind their specific solute and undergo a series
○ Molecular catabolism of conformational changes to transfer the ligand across the
○ Energy generation membrane; relatively slow transport
● Endocytosis – uptake of fluids or macromolecules by the cell
B. CELL MEMBRANE
○ Caveolae-mediated Endocytosis
● Provides selective, structural barrier between the cell and the
■ Caveolae – non-coated plasma membrane invaginations
outside world.
associated with GPI-linked molecules, cyclic adenosine
○ Think of the cell membrane as an international boundary where
monophosphate (cAMP) binding proteins, src-family kinases,
customs officers keep watch of traffic flow and determine what is
and the folate receptor; regulate transmembrane signaling and
allowed to cross back and forth. cellular adhesion via internalization of receptors and integrins
● Associated with receptors and transport proteins ■ Caveolin – major structural protein of caveolae, enriched in
○ The customs officers are your receptor proteins which detect glycosphingolipids and cholesterol
signals from the outside world; transport proteins help some ■ Potocytosis – “cellular sipping”; Internalization of caveolae
molecules get across a membrane. along with bound molecules and associated extracellular fluid.
● Composed of an amphipathic phospholipid bilayer containing: ○ Receptor-mediated Endocytosis
Integral membrane proteins ■ Macromolecules bound to membrane receptors are taken up at
Peripheral membrane proteins specialized regions of the plasma membrane called
Cholesterol clathrin-coated pits
● Amphipathic- having both hydrophilic and hydrophobic parts ■ Receptors are efficiently internalized by membrane
invaginations driven by the associated clathrin matrix,
eventually pinching off to form clathrin coated vesicles
■ The vesicles then rapidly lose their clathrin coating and fuse
with an acidic intracellular structure called the early endosome
■ The endosomal vesicles undergo progressive maturation to late
endosomes, ultimately fusing with lysosomes
○ Pinocytosis – “cellular drinking”
○ Phagocytosis – “cellular eating”; macrophages and neutrophils
● Exocytosis – method by which macromolecules are exported from
cells (e.g., exocytosis of insulin)
● Transcytosis – movement of endocytosed vesicles between the
apical and basolateral compartments

C. CYTOPLASM
● Inclusions: cytosol, plasma membrane, organelles
● A fluid filled interior packed with molecules, structures and lots of
activities.
● Imagine it being a crowded party filled with people and
conversations.
● Many of the chemical reactions that make up the metabolism of the
cell happens in cytoplasm.
Figure 9. Cell Membrane ● Molecules and cellular components are on a constant move being
transported from one place to another and just moving randomly due
to their kinetic energy.
Modified Fluid-Mosaic Model
● Fatty-acid chains of the lipid molecules face each other, making the D. MITOCHONDRIA
inner portion of the membrane hydrophobic (i.e., having no affinity ● Fire place of the cell
for water). ● This is where eukaryotes, like ourselves, burn food by respiration.
● Surfaces of the membrane are formed by the polar head groups of They help power up the cell by transferring energy from food to ATP.
the lipid molecules, thereby making the surfaces hydrophilic (i.e.,
they have an affinity for water).
● Composed of an outer and an inner membrane with an intervening
compartment in between (intermembrane space)

4
● Inner membrane is folded to form flat, shelf -like structures (cristae)
and encloses a viscous fluid - filled space (matrix space)
● Contain their own DNA that encodes about 1% of total cellular
protein and approx. 20% of the proteins involved in oxidative
phosphorylation
● Functions:
○ Aerobic energy supply (oxidative phosphorylation, ATP)
○ Initiation of cell death (apoptosis)
○ Necrosis
○ Can carry out all the steps of DNA replication, transcription, and
translation
● Energy Generation:
○ Each mitochondrion has two separate membranes with distinct
functions:
■ Inner membrane contains the enzymes of the respiratory chain
■ Intermembrane space site of nucleotide phosphorylation
■ Outer membrane forms voltage-dependent anion channels
○ The major source of the energy that facilitates all cellular function
derives from oxidative metabolism.
○ Important source of reactive oxygen species Figure 11. Ribosome
● Intermediate Metabolism: Oxidative Phosphorylation
○ Yields 36-38 ATP F. ENDOPLASMIC RETICULUM
○ This process is expensive and uses all carbon moieties that no ● Composed of a network of tubules, sacs, and flat sheets of
carbon is left to use for building lipids and proteins membranes that occupy intracellular space
● Warburg Effect ● Help move substances within the cell
○ To ensure that there will be Carbon available left for building other ● Site for synthesis of all transmembrane lipids for
macromolecules, it allows a glucose molecule to undergo plasma membrane and lipids for plasma membrane
anaerobic metabolism yielding lactic acid and 2 ATP but leaving and cellular organelles
intermediates to be used for lipids, amino acids and nucleic acids. ● Composed of contiguous but distinct domains that are
● Cell Death: Necrosis: distinguished by the presence of ribosomes; Rough
○ External cellular injury can damage mitochondria creating pores in ER or Smooth ER
the outer membrane resulting to cell death ● 2 types
● Apoptosis ○ Smooth ER
○ Integrate intracellular proapoptotic and antiapoptotic effector ■ Functions in the synthesis of cholesterols
signals to generate a final “go” or “no go” signal for apoptosis. and lipids
■ Functions in the detoxification of certain
drugs and toxins (Ex. barbiturates and
alcohol)
■ In skeletal muscle cells, sequesters and
releases calcium ions thus regulating
muscle contraction and relaxation
(sarcoplasmic reticulum)
○ Sarcoplasmic Reticulum - Responsible for the cyclic release
and sequestration of calcium ions that regulate muscle
contraction and relaxation
■ Exists as the transition zone to generate transport vesicles that
carry newly synthesized proteins to the Golgi Apparatus
■ Responsible for sequestering intracellular Calcium
○ Rough ER
■ Continuous with outer nuclear membrane
■ Contains surface possesses receptor molecules for ribosomes
(ribophorins) and signal recognition particles (SRPs, docking
protein)
■ Binds ribosomes engaged in translating mRNA for proteins
destined for secretion or for membrane insertion (“by placing a
tag”)
■ May modify of proteins and membrane lipid synthesis
■ Membrane bound ribosomes on the systolic face of RER:
Figure 10. Mitochondria
translates mRNA into proteins
■ Synthesis of proteins with signal peptides is initiated on free
E. RIBOSOME
ribosomes
● Small, bipartite, non-membranous organelles
■ Detection of excess abnormally folded proteins -> reduction in
● Can be seen floating free or attached to endoplasmic reticulum
protein synthesis; failure to correct overload -> cell death
● 2 subunits:
1. Larger 60s (apoptosis)
2. Smaller 40s
● Each subunit is composed of proteins and rRNA G. GOLGI APPARATUS
● Act as “workbench” that provides a surface upon which protein ● Composed of a specifically oriented cluster of
synthesis occurs vesicles, tubules, and flattened membrane-bounded
● Facilitates the synthesis of proteins by translating protein coding cisternae
sequence from mRNA ● Site of post-translational modifications and sorting of
● Ribosomal sizes are measured by Svedberg units. This unit newly synthesized proteins and lipids
describes how fast particles fall-out of solutions during ● Modify proteins from cis (near the ER) to trans (near
centrifugation. the plasma membrane)
● The reason why antibiotics work is because they can target bacteria ● Each complex has:
based on ribosomal size. ○ Cis face: convex entry face, close to the nucleus
○ Tetracycline and Aminoglycosides are protein synthesis inhibitors ○ Trans face: concave exit face, oriented toward the cell membrane
targeting 30s. ○ Medial face: intermediate cisternae between cis face and trans
○ Chloramphenicol and Erythromycin attack 50s. face

5
● Functions:
○ Modifies of proteins
○ Sorts and packs molecules for secretion or transport to other
organelles
● Trans Golgi network – proteins and lipids are sorted and dispatched
to other organelles, plasma membrane, or secretory vesicles.
● Golgi complex – prominent in cells specialized for secretion

FIGURE 12. Perinuclear Hoff in malignant plasma cells.

● In plasma cells that secrete antibodies, the golgi can be recognized


as a perinuclear hof on simple hematoxylin and eosin stains.
● I-CELL DISEASE (MUCOLIPIDOSES TYPE 2) FIGURE 14. Pathogenesis of I-cell disease
○ Defective sorting, packing, or shipping will result in Inclusion Cell
(I-Cell) Disease and Hyperinsulinemia.
H. ENDOSOMES
○ Part of a family of lysosomal storage diseases which results in
● Intermediate compartments within the cell.
defective phosphotransferase, an enzyme found in the Golgi
● Membrane-bound vesicles formed by a complex family of processes
apparatus, which transfers phosphate to mannose residues on
collectively known as endocytosis.
specific proteins to be targeted to lysosomes
● Used in the destruction and transport of endocytosed,
○ Clinical Presentation
phagocytosed, or autophagocytosed materials.
■ Abnormal skeletal development
● Possess proton pumps in their membranes, which pump H+ into the
■ Coarse facial features
endosome, thus acidifying the interior of this compartment.
■ Restricted joint movement
■ Die before the age of 7 due to congestive heart failure
■ Recurrent respiratory infections I. LYSOSOMES
● Late endosomes
● Contain digestive enzymes
● Functions
○ Aid in cell renewal
○ Break down old cell parts
○ Digests invaders
● “Cell Recycling Center”
● Lysosomal Storage Disorders - lysosomal disorders are triggered
when a particular enzyme exists in too small an amount or is missing
● When this happens, substances accumulate in the cell
● Have two forms:
○ Primary Lysosomes – do not have the materials
to be digested yet and therefore make bonding on the trans side
of the golgi.
○ Secondary Lysosomes – formed by fusion of the primary
lysosomes with the substrate to be degraded.
● Phagolysosomes – phagocytosed particulate matter
● Phagocytosis – material is engulfed to form a phagosome that
subsequently fuses with lysosomes
● Autophagolysomes – autophagocytosed material
● Residual bodies – indigestible material
FIGURE 13. Child with I-cell disease ● Lysosomal enzymes are initially synthesized in the ER lumen and
then tagged with mannose-6-phosphate (M6P) within the Golgi
apparatus
○ M6P-modified proteins – subsequently delivered to lysosomes
through trans Golgi vesicles
Note: See Appendix C
● Lysosomal storage disorders are caused by
lysosomal dysfunction usually as a consequence of
deficiency of a single enzyme required for metabolism
of lipids, glycoproteins, mucopolysaccharides.
● GAUCHER DISEASE
○ Most common lysosomal storage disease, which is a deficiency
due to an enzyme glucocerebrosidase resulting in product
accumulation of glucocerebrosides.

6
○ Clinical Presentations: liver enlargement due to macrophage L. CYTOSKELETON
accumulation (wrinkled tissue paper or crumpled paper ● Composed of a filamentous array of proteins
appearance of the cytoplasm),Gaucher cells accumulation in ● Structural framework of cell
the bone marrow resulting in infarction, aseptic necrosis of
● Transport material from one region of the cell to another and provide
femur,or bone crisis
it with the capability of motion and cell division
● Cytoskeletal structures are constantly elongating and shrinking
● Microfilaments and microtubules are most active, and their
assembly and disassembly can drive cell migration
● Components:
○ Microtubules
■ 25-nm thick fibrils
■ Heterodimer composed of α and β tubulin molecules
○ These fibrils are extremely dynamic and polarized with “+”
and “-” ends
○ “-” end is typically embedded in a microtubule organizing
center or centrosome near the nucleus
○ “+” end elongates r recedes in response to various stimuli
by the addition or subtraction of tubulin dimers
■ Implicated in intracellular transport of organelles and vesicles
FIGURE 15. Gaucher Disease ■ Provide basis for several complex cytoplasmic components, ex.
centrioles, basal bodies, cilia, flagella
J. PEROXISOMES ○ Microfilaments (thin actin filaments)
● Membrane-bounded organelles housing oxidative enzymes ■ Actin is present in muscle as thin (5-7 nm diameter) filament
○ Ex. urate oxidase, D - amino acid oxidase, catalase
composed of globular subunits organized into double-stranded
● All peroxisomes are formed by fission from preexisting peroxisomes
helix
● Functions:
■ In most cells, they are found scattered in unorganized fashion
○ Formation of free radicals (e.g., superoxide) which destroy
various substances ■ Most activities depend on interaction with myosin (a thick
■ Using molecular oxygen, peroxisome enzymes remove filament)
hydrogen atoms from specific organic substrates, and in an ■ Two types of movement:
oxidative reaction, produce hydrogen peroxide (toxic) ○ Local movement
○ Protection of cell by degrading hydrogen peroxide by catalase ○ Sliding movement
■ Globular protein actin (G-actin)
○ Detoxification of certain toxins
○ The most abundant cytosolic protein in cells
○ Elongation of some fatty acids during lipid synthesis ○ Intermediate filaments
● Peroxisomes in the liver are good in breaking down toxins such as
■ 10-12 nm diameter
ethanol and alcoholic beverages.
■ Function in providing a structural framework to the cell and
● Problems in peroxisomes will have problems in oxidizing long chain
resisting mechanical stresses placed on cells
fatty acids.
■ Includes keratin proteins and nuclear lamins
■ Form ropelike polymers and do not usually actively reorganize
K. PROTEASOMES
○ Thick filaments
● Small, barrel-shaped organelles
● Function in the degradation of cytosolic proteins
○ These include denatured or misfolded proteins VI. CELLULAR COMMUNICATION
○ Digest proteins into small (6 to 12 amino acids) fragments A. CELL SIGNALING
● Proteasomal degradation of regulatory proteins or transcription ● Many cells require certain input to continue living.
● In the absence of appropriate exogenous signals, they die by
factors can trigger initiation or suppression of signaling pathways.
apoptosis.
● Destruction of damaged and old proteins by tagging with ubiquitin
● The signals that most cells respond to can be classified into several
● 2 types:
groups:
○ Larger 26s
○ Danger and pathogens – damage to neighboring cells and
○ Smaller 20s
pathogens. Many cells have an innate capacity to sense and
respond to damaged cells (danger signals), as well as foreign
invaders such as microbes.
○ Cell-cell contacts – contact with neighboring cells, mediated
through adhesion molecules and/or gap junctions.
○ Cell-ECM contacts – mediated through integrins
○ Secreted molecules.
■ Cytokines, a term reserved for mediators of inflammation and
immune responses
■ Hormones, which are secreted by endocrine organs and act on
different cell types
■ The most important secreted molecules include growth factors
● Spatial relationships between the sending and receiving cells:
○ Paracrine signaling – only cells in the immediate vicinity are
affected
Figure 16. Proteasomes ○ Autocrine signaling – cell affect the same cell

7
○ Synaptic signaling – activated neurons secrete neurotransmitter
at specialized cell junctions onto target cells
○ Endocrine signaling – a mediator is released into the
bloodstream and acts on target cells at a distance
● Regardless of the nature of an extracellular stimulus (paracrine,
autocrine, synaptic, or endocrine) the signal that is conveyed is
transmitted to the cell via specific receptor proteins
● Cell signaling specific receptors:
○ Intracellular Receptors
○ Cell Surface Receptors

INTRACELLULAR RECEPTORS
● Transcription factors activated by lipid-soluble ligands that can easily
cross the plasma membrane
● Includes receptors for Vit D and steroid hormones

CELL SURFACE RECEPTORS


● Generally transmembrane proteins with extracellular domains that
bind soluble secreted ligands
● Depending on the receptor, ligand binding can:
○ Open ion channels, typically at the synapse between electrically
excitable cells
○ Activate an associated GTP-binding regulatory protein (G
protein)
○ Activate an endogenous or associated enzyme (often a tyrosine
kinase)
○ Trigger a proteolytic event or change protein binding or stability to
Figure 17. Signal Transduction: G-protein coupled receptor
activate a latent transcription factor
NUCLEAR RECEPTORS
B. SIGNAL TRANSDUCTION
● Cellular receptors are grouped into several types based on the ● These are class of proteins found inside the cell and respond to
signaling mechanisms they use and the intracellular biochemical signals that pass through the plasma membrane
pathways they activate are: ● They can bind directly to DNA and regulate the expression of nearby
○ Receptors associated with kinase activity genes; the term transcription factors or sequence specific DNA
○ G-protein coupled receptors binding factors
○ Nuclear receptors ● Alone with or with other proteins their job is to either promote or
block RNA polymerase
TYROSINE KINASE SIGNALING ○ RNA polymerase – the enzyme that copies genetic instructions
from DNA to RNA for specific genes.
● Signal transduction using this type of receptor involves the following ● Lipid soluble ligands can diffuse into cells where they interact with
steps: intracellular proteins to form a receptor-ligand complex that directly
○ The primary messenger binds the receptor (which is binds to DNA
phosphorylated) and becomes an active enzyme. ● The results can either be activation or repression of gene
○ The receptor activates another membrane associated protein transcription
called RAS Note: See Appendix E
○ And finally, the RAS kicks off the phosphorylation cascade
● Downstream phosphorylation is a common pathway of signal OTHER CLASSES OF RECEPTOR
transduction.
● Changes in receptor geometry can stimulate intrinsic receptor ● Originally recognized as important for embryonic development and
protein kinase activity or remote the enzymatic activity of recruited cell fate determination
intracellular kinases ● Participate in the functioning of mature cells,particularly within the
Note: See Appendix D immune system
● They rely on protein-protein interactions to transduce signals
G-PROTEIN SIGNALING
C. TRANSCRIPTION FACTORS
● They are called g-proteins because they bind to GDP. ● Most signal transduction pathways ultimately induce durable effects
● The signal binds to the satellite dish, then it activates its G-protein. on cellular function by modulating gene transcription; this occurs
● Part of the G-protein travels along the membrane to bind to the through the activation and/or nuclear localization of transcription
enzyme (adenylyl cyclase) factors.
● The enzyme catalyzes the production of secondary messenger ● Some transcription factors drive expression of a relatively limited set
molecules (activation) of genes or a specific genetic program, while others have
● However, the signal must stop at one point. widespread effects.
● Signal transduction pathways are shut down once the primary ● Among the transcription factors that regulate cell division are
messenger stops sending signals. products of several growth-promoting genes, such as MYC and JUN
● To stop signal transduction, cells need to deactivate the molecules ● And of and of cell cycle–inhibiting genes, such as TP53
that are involved with the pathway ● Transcription factors often contain modular domains that bind to
● Traverse the plasma membrane seven times DNA, small molecules such as steroid hormones, and intracellular
● After ligand binding, the GPCR associates with an intracellular regulatory proteins.
guanosine triphosphate binding G protein that contains GDP ● Interactions mediated by these domains can be controlled by
posttranslational modifications such as phosphorylation.
● These changes can result in translocation from the cytoplasm into
the nucleus, modify transcription factor protein half-life, expose
specific DNA binding motifs, or promote binding to components of
the RNA polymerase complex to augment transcription factor activity

8
DNA-BINDING DOMAINS ● Acts as a morphogen in embryonic development (i.e., influences the
pattern of tissue differentiation)
● Permit specific binding to short DNA sequences. Whereas some ● Promotes cell migration (hence the designation scatter factor)
transcription factor binding sites are found in promoters near the ● Enhances hepatocyte survival.
location of transcription initiation, other transcription factor binding ● Produced by fibroblasts and most mesenchymal cells, endothelial
sites can be found throughout the genome; cells, and non hepatocyte liver cells.
● In the latter case, transcription factor activation may lead to the ● Synthesized as an inactive precursor (pro-HGF) that is
simultaneous transcription of a cassette of genes (presumably proteolytically activated by serine proteases released at sites of
interrelated and interacting). injury.
● Transcription factors may also bind to long-range regulatory ● MET - receptor of HGF
elements such as enhancers that function by bringing gene ○ Has intrinsic tyrosine kinase activity.
promoters into geographic proximity to the genes they regulate ○ Frequently overexpressed or mutated in tumors, particularly renal
● The fact that these sites may be distant from one another based on and thyroid papillary carcinomas.
the linear genetic sequence emphasizes the importance of ○ MET inhibitors are being evaluated for cancer therapy.
chromatin organization in regulating gene expression

PROTEIN-PROTEIN INTERACTION DOMAINS PLATELET-DERIVED GROWTH FACTOR (PDGF)


● Family of several closely related proteins, each consisting of two
● Within transcription factors directly or indirectly recruit additional chains (designated by pairs of letters).
proteins including coactivators, histone-modifying enzymes, and ● Three isoforms of PDGF (AA, AB, and BB) are directly biologically
chromatin-remodeling complexes that unwind and/or otherwise active; PDGF-CC and PDGF-DD must be activated by proteolytic
expose initiation sites. cleavage.
● Most importantly, they recruit RNA polymerase — the large ● PDGF proteins are stored in cytoplasmic granules and released by
multiprotein enzymatic complex that is responsible for RNA activated platelets.
synthesis. ● Although originally isolated from platelets (hence the name), PDGFs
are produced by many cells including activated macrophages,
D. GROWTH FACTORS AND RECEPTORS endothelium, smooth muscle cells, and tumors.
● Growth factors stimulate the activity of signaling pathways and ● All PDGF isoforms exert their effects by binding to two cell-surface
genes that augment cell survival, growth, and division. receptors (PDGFR α and β), both of which have intrinsic tyrosine
● Growth factors bind to specific receptors and, ultimately, influence kinase activity.
expression of genes that: ● Induces fibroblast, endothelial, and smooth muscle cell proliferation
○ Promote entry into the cell cycle. and is also chemotactic for these cells (and inflammatory cells),
○ Relieve blocks on cell cycle progression (thus promoting thereby promoting their recruitment to sites of inflammation and
replication). tissue injury
○ Prevent apoptosis.
○ Enhance synthesis of components (nucleic acids, proteins, lipids,
VASCULAR ENDOTHELIAL GROWTH FACTOR (VEGF).
carbohydrates) required for cell division.
● Although growth factors are characteristically thought of as proteins ● Are a family of homodimeric proteins
that "just" stimulate cell proliferation and/or survival, it is important to ○ VEGF-A
remember that they can also regulate a host of non-growth activities ■ Generally referred to simply as VEGF
including migration, differentiation, and synthetic capacity. ■ It is the major angiogenic factor (inducing blood vessel
● Growth factors regulate cellular proliferation at steady state and in development) after injury and in tumors
response to injury, when irreversibly damaged cells must be ○ VEGF-B
replaced. ■ Together with PIGF, are involved in embryonic vessel
● Uncontrolled proliferation can result when the growth factor activity development,
is dysregulated or when growth factor signaling pathways are ○ VEGF-C
altered to become constitutively active. ■ Together with VEG-D, stimulate both angiogenesis and
● Thus many growth factor pathway genes are proto-oncogenes; by lymphatic development (lymphangiogenesis).
virtue of their proliferative effects, gain-of-function mutations convert ○ VEGF-D
them into oncogenes that lead to unfettered cell division and can be ○ Placental growth factor (PlGF)
precursors to malignancy. ● Involved in the maintenance of normal endothelium, with highest
● Although the growth factors described here all involve receptors with expression in epithelial cells adjacent to fenestrated epithelium (e.g.,
intrinsic kinase activity, growth factors can signal through each of the kidney podocytes, retinal pigment epithelium, and choroid plexus).
pathways. ● Induces all the activities necessary for angiogenesis, including
endothelial cell migration and proliferation (capillary sprouting), and
promotes formation of vascular lumina.
EPIDERMAL GROWTH FACTOR (EGF) & TRANSFORMING
● Affects vascular dilation and increases vascular permeability (VEGF
GROWTH FACTOR-α (TGF-α)
was originally called vascular permeability factor to reflect that
● Both belong to the EGF family, bind to overlapping sets of receptors, activity).
and share many biologic activities. ● Hypoxia
● EGF and TGF-α, which are produced by macrophages and some ○ Most important inducer of VEGF production, through pathways
epithelial cells, are mitogenic for hepatocytes, fibroblasts, and a host that involve activation of the transcription factor hypoxia-inducible
of epithelial cell types. factor 1 (HIF-1). Other VEGF inducers—produced at sites of
● The "EGF receptor family" includes four membrane receptors with inflammation or wound healing—include PDGF and TGF-α.
intrinsic tyrosine kinase activity; the best-characterized receptor is ● Bind to a family of tyrosine kinase receptors:
EGFR1, also known as ERB-B1, or simply EGFR. ○ VEGFR-1
○ EGFR1 mutations and/or amplification frequently occur in a ○ VEGFR-2
number of cancers including lung, head and neck, breast, and ■ Highly expressed in endothelium and is the most important for
brain. angiogenesis.
● ERB-B2 receptor (also known as HER-2) is overexpressed in a ○ VEGFR-3
subset of breast cancers. ● Antibodies against VEGF are approved for the treatment of tumors
● Antibodies and small molecule antagonists that target many of these such as renal and colon cancers that require angiogenesis for their
receptors have proven effective in some cancers. spread and growth.
● Anti-VEGF therapies have had success in ophthalmic disorders
HEPATOCYTE GROWTH FACTOR (HGF) including “wet” age-related macular degeneration (a disorder of
● “Scatter Factor” inappropriate angiogenesis and vascular permeability that causes
● Has mitogenic effects on hepatocytes and most epithelium including adult-onset blindness), angiogenesis associated with retinopathy of
biliary, lung, kidney, breast, and skin. prematurity, and vascular leakage that leads to diabetic macular
edema.

9
● Increased levels of soluble VEGFR-1 (also known as s-FLT-1) in ■ The major nonfluid constituents of the interstitial matrix are
pregnant women may cause preeclampsia (hypertension and fibrillar and non fibrillar collagens, as well as fibronectin, elastin,
proteinuria) by sequestering the free VEGF required for proteoglycans, hyaluronate, and other constituents.
maintenance of normal endothelium. ■ Synthesized by mesenchymal cells
● Fibroblasts
FIBROBLAST GROWTH FACTOR (FGF) ○ Basement membrane
● Refers to a family of growth factors with more than 20 members. ■ Synthesized by the overlying epithelium and underlying
○ Acidic FGF (aFGF) (also known as FGF-1) and basic FGF mesenchymal cells, form a flat lamellar mesh.
(bFGF) (also known as FGF-2) are the best characterized ■ The major constituents are non fibrillar typeIV collagen and
○ FGF-7 is also referred to as keratinocyte growth factor (KGF). laminin.
● Released FGFs associate with heparan sulfate in the ECM, which ■ Specialized surfaces for cell growth
serves as a reservoir for inactive factors that can be subsequently ● Components of ECM:
released by proteolysis (e.g., at sites of wound healing). ○ Fibrous structural proteins
● FGFs signal through four tyrosine kinase receptors (FGFR1 through ■ Such as collagens and elastin that confer tensile strength and
FGFR4) to promote wound healing, hematopoiesis, and recoil
development; bFGF has all the activities necessary for ○ Water-hydrated gels
angiogenesis. ■ Such as proteoglycans and hyaluronan that permit
compressive resistance and lubrication.
○ Adhesive glycoproteins
TRANSFORMING GROWTH FACTOR β (TGF-β) ■ Connect ECM elements to one another and to cells.
● Has three isoforms (TGF-β1, TGF-β2, TGF-β3) that belong to a
larger family of about 30 members including bone morphogenetic
proteins (BMPs), activins, inhibins, and Müllerian inhibiting
substance.
○ TGF-β1
■ Has the most widespread distribution and is commonly referred
to simply as TGF-β
■ A homodimeric protein produced by multiple cell types
including platelets, endothelium, epithelial cells, and
inflammatory cells.
● Is secreted as a precursor that requires proteolysis to yield the
biologically active protein.
● There are two TGF-β receptors (types I and II) both with
serine/threonine kinase activity that induce the phosphorylation of a
variety of downstream transcription factors called Smads.
● Phosphorylated Smads form heterodimers, allowing nuclear
translocation and association with other DNA-binding proteins to
activate or inhibit gene transcription. Figure 18. Illustration depicting extracellular matrix in relation to epithelium,
● TGF-β signaling has multiple—and often opposing—effects, endothelium & connective tissue.
depending on the tissue type and any concurrent signals.
● Agents with such multiplicity of effects are called pleiotropic, and
TGF-β is “pleiotropic with a vengeance.”
VII. CELL CYCLE
● TGF-β can be conceptualized as driving scar formation and putting a ● Cell proliferation is fundamental to development, maintenance of
brake on the inflammation that accompanies wound healing. steady-state tissue homeostasis, and to replacement of dead or
○ TGF-β stimulates the production of collagen, fibronectin, and damaged cells.
proteoglycans and inhibits collagen degradation by both ● Key elements:
decreasing matrix metalloproteinase (MMP) activity and ○ Accurate DNA replication
increasing the activity of tissue inhibitors of proteinases (TIMPs) ○ Coordinated synthesis of all other cellular components
(discussed later). ○ Equal apportionment of DNA and organelles to daughter cells
○ Involved not only in scar formation after injury but also drives through mitosis and cytokinesis
fibrosis in lung, liver, intestines, and kidneys in the setting of ● The cell cycle is a series of events that take place in a cell leading to
chronic inflammation. its division and duplication of its DNA to produce two daughter cells.
○ TGF-β is also an antiinflammatory cytokine that serves to limit and It consists of:
terminate inflammatory responses. ○ G1 (gap 1) – actively functioning cells; grow and copy all their
■ It does this by inhibiting lymphocyte proliferation and activity contents except DNA
of other leukocytes. ○ S (Synthesis) – cells copy DNA by DNA replication
■ Animals lacking TGF-β have widespread and persistent ○ G2 (gap 2) – checkpoint
inflammation. ○ M (mitotic phase) – divides chromosomes
Note: See Appendix F ● Quiescent cells that are not actively cycling are in the G0 (gap 0)
state (Nerve and Skeletal muscle cells).
● Cells can enter G1 either from the G0 or after completing a round of
E. EXTRACELLULAR MATRIX (ECM) mitosis.
● A network of interstitial proteins that constitutes a significant ● Each stage requires completion of the previous step, as well as
proportion of any tissue activation of necessary factors.
● Cell interactions are critical for development and healing, as well as ● Non-fidelity of DNA replication or cofactor deficiency results in arrest
for maintaining normal tissue architecture at various transition points.
● Key functions: Note: See Appendix G.
○ Mechanical support
○ Control of cell proliferation
○ Scaffolding for tissue renewal
A. CHECKPOINT
○ Establishment of tissue microenvironments ● Checkpoints are points in the cell cycle where checks are made to
● ECM occurs in two basic forms: make sure that everything is proceeding normally.
○ Interstitial matrix ● If the damage is beyond repair the cell dies by suicide.
■ Occupies the spaces between stromal cells within connective ● Quality control checkpoints are surveillance mechanism primed to
tissue and between parenchymal epithelium and the underlying sense DNA or chromosomal damage
supportive vascular and smooth muscle structures in some ● These quality control checkpoints ensure that cells with genetic
organs. imperfections do not complete replication.
1. G1-S checkpoint – monitors DNA integrity before irreversibly
committing cellular resources to DNA replication.

10
2. G2-M restriction point – ensures that there has been accurate 2. p15 (CDKN2B), p16 (CDKN2A), p18 (CDKN2C), p19 (CDKN2D)
genetic replication before the cell actually divides. - Selective effects on cyclin CDK4 and CDK6.
3. Mitosis checkpoint 3. Defective CDKI checkpoint proteins - Allow cells with damaged
● When cells do detect DNA irregularities, checkpoint activation delays DNA to divide, resulting in mutated daughter cells at risk for
cell cycle progression and triggers DNA repair mechanisms. malignant transformation
● If the genetic derangement is too severe to be repaired, cells either
undergo apoptosis or enter a nonreplicative state called senescence
– primarily through p53-dependent mechanisms.

B. CYCLINS & CYCLIN-DEPENDENT KINASES


● The decision to progress or to stop a cell cycle depends on the
action of molecular regulators called cyclin-dependent kinases.
● The cell cycle is regulated by activators and inhibitors.
● More than 15 cyclins have been identified; cyclins D, E, A, and B
appear sequentially during the cell cycle and bind to one or more
CDKs.
● CDK Inhibitors
○ Enforcing the cell cycle checkpoints.
○ They accomplish this by modulating CDKcyclin complex activity.
● There are several different CDKIs.
○ One family of CDKIs – Composed of three proteins called p21
(CDKN1A), p27 (CDKN1B), and p57 (CDKN1C) – Broadly inhibits
multiple CDKs.
○ Another family of CDKIs has selective effects on cyclin CDK4 and
cyclin CDK6; these proteins are called p15 (CDKN2B), p16
(CDKN2A), p18 (CDKN2C), and p19 (CDKN2D).
● Defective CDKI checkpoint proteins
○ Allow cells with damaged DNA to divide, resulting in mutated
daughter cells at risk for malignant transformation.
● Cyclins are proteins that signal the cell to divide
● Each cyclin works only in specific phases of the cell cycle at specific
times
○ Cyclin D – G1-S phase
○ Cyclin E CDK
○ Cyclin A CDK Figure 20. Role of cyclins, cyclin-dependent kinases (CDKs) and CDK inhibitors
○ Cyclin D CDK (CDKIs) in regulating the cell cycle.
● At the checkpoint the cell finds a damaged DNA, so this is where the
suppressors come into play (p53, p21, p10). VIII. STEM CELL
● G1 phase is arrested by p53, so it buys the cell time to fix the DNA. ● Stem cells have the dual property of being able to self-renew and to
● If the damage of the DNAse is fixed, p53 would be degraded. If the give rise to differentiated cells and tissues.
DNA repair fails, the cell permanently arrests and undergo apoptosis ● Stem cells are cells that can give rise to nearly all our tissues.
(program cell death). ● Totipotent stem cells give rise to a full range of differentiated tissues
● When p53 stops the cell from dividing, an increase in p53 levels during development.
leads to an increase in the production of p21. ● In mature organisms, adult stem cells have the capacity to replace
● p21 blocks Cyclin CDK complexes which in turn leads to damaged cells and maintain cell populations within tissues where
hypo-phosphorylation of the RB gene they reside.
● This causes the cell to arrest at G1 phase to S phase. ● Under conditions of homeostasis, stem cells are maintained by
self-renewal, which can involve two types of cell division:
○ Asymmetric division
■ Cell replication in which one daughter cell enters a
differentiation pathway and gives rise to mature cells, while the
other remains undifferentiated and retains its self-renewal
capacity.
○ Symmetric division
■ When both daughter cells retain self-renewal capacity.
■ Such replication occurs early in embryogenesis – when stem
cell populations are expanding and under stress conditions, as
in bone marrow repopulation after ablative chemotherapy.
● Key features:
○ Self-renewal – they can reproduce themselves for long periods
○ Differentiation and specialization – under certain conditions in the
body or the laboratory produce cells that eventually become
specific types of cells.
● Two broad types:
○ Embryonic stem cell – can give rise to any type of cell in the
human body
■ The most undifferentiated
■ Present in the inner cell mass of the blastocyst
■ Have virtually limitless cell renewal capacity
Figure 19. Checkpoint in damaged DNA
■ Said to be totipotent
■ Can be induced under appropriate culture conditions to form
● The role of mdm2 protein is to act as a key negative regulator of specialized cells of all three germ layers.
p53. ○ Adult stem cell (a.k.a. tissue stem cell) – can make any type of
● Enforcing the cell cycle checkpoints is the job of CDK inhibitors cell of that particular tissue
(CDKIs) by modulating CDK-cyclin complex activity. There are ■ Found in intimate association with the differentiated cells of a
several kinds of CDKIs: given tissue.
1. p21 (CDKN1A), p27 (CDKN1B) and p57 (CDKN1C) - Inhibits
multiple CDKIs.

11
■ Normally protected within specialized tissue microenvironments REFERENCES
called stem cell niches. ● Kumar, V., Abbas, A. K., Aster, J. C., & Perkins, J. A. (2018). Robbins Basic
● Niches - found in many organs like the bone marrow, Pathology (Tenth edition.). Philadelphia: Elsevier.
intestines where epithelial stem cells are confined to the ● 2024. 1.01 Cell: As A Unit of Health and Disease
crypts, bulge region of hair follicles, limbus of cornea and
sub ventricular zone in the brain
● Can usually produce only those cells normally found in tissue
they reside
● Limited range (lineage potential) of differentiated cells they
can generate
● Protipotent Stem Cell
○ Can practically turn into a whole organism
● Chloropotent Stem Cell
○ Can give rise to all of the cell types that form human body
● Multipotent Stem Cell
○ Give rise to cells of certain category
● Totipotent Stem Cell
○ Can give rise to the full range of differentiated tissues; in the
mature organism.
● Adult Stem Cell
○ Only have the capacity to replace damaged cells and maintain
cell populations within the tissues where they reside.
○ Although adult stem cells can maintain tissues with high (e.g.,
skin, and gastrointestinal tract) or low (e.g., endothelial) cell
turnover, the adult stem cells in any given tissue can usually
produce only those cells normally found within that tissue.
● Hematopoietic Stem Cell
○ Continuously replenish all the cellular elements of the blood as
they exit the circulation, senesce, or are otherwise consumed.
○ Can be isolated directly from bone marrow, as well as from the
peripheral blood after administration of certain colony-stimulating
factors that induce their release from bone marrow niches.
○ They are overall rare.
○ Clinically, these stem cells can be used to repopulate marrows
depleted after chemotherapy (e.g., for leukemia) or to provide
normal precursors to correct various blood cell defects (e.g.,
sickle cell disease).
● Mesenchymal Stem Cell
○ Multipotent cells that can differentiate into a variety of stromal
cells including chondrocytes (cartilage), osteocytes (bone),
adipocytes (fat), and myocytes (muscle).
○ These cells can be massively expanded and can generate a
locally immunosuppressive microenvironment.
○ Represent a potential means of manufacturing stromal cellular
scaffoldings for tissue regeneration.

12
APPENDIX

APPENDIX A. Copy Number Variation

APPENDIX B. DNA Organization

13
Appendix C. Lysosomal Storage Disorders

Appendix D. Signal Transduction

14
Appendix E. Nuclear Receptor

APPENDIX F. Growth Factors Involved in Regeneration and Repair

15
APPENDIX G. Cell Cycle Landmarks

16
LE 1.02 CELLULAR RESPONSES TO STRESS:
ADAPTATION, INJURY AND DEATH
September 05, 2022
○ Cells can only adapt up to a certain point when the stress is so
OUTLINE severe the cells can’t adapt (Irreversible Injury).
I. Introduction B. Irreversible Injury ○ Excessive cell death as a result of progressive injury is one of the
II. Overview of Cellular V. Necrosis most crucial events in the evolution of disease in any tissue or
Responses to Stress and A. Coagulative Necrosis organ which is a result of diverse causes such as ischemia
Noxious Stimuli B. Liquefactive Necrosis (inadequate blood flow), infections, toxins, and immune reactions.
A. Causes of Cell Injury C. Enzymatic Fat Necrosis ● 2 Principal Pathways of Cell Death:
B. Progression of Cell Injury D. Traumatic Fat Necrosis ○ Necrosis
and Death E. Caseous Necrosis ○ Apoptosis
III. Mechanisms of Cell Injury F. Fibrinoid Necrosis ● Cellular responses to injury depends on the following:
A. ATP Depletion G. Gangrenous Necrosis
○ Type of Injury
B. Mitochondrial Damage VI. Apoptosis
○ Duration of injury (including pattern)
C. Influx of Calcium and Loss A. Causes of Apoptosis
of Calcium Homeostasis B. Mechanisms of Apoptosis ○ Severity and Intensity of injury
D. Accumulation of C. Other Pathways of Cell ○ Type of cell injured
Oxygen-Derived Free Death ○ Cell's metabolic state
Radicals (Oxidative VII. Autophagy ○ Cell's ability to adapt
Stress) VIII. Adaptation of Cellular ● Critical intracellular systems susceptible to injury:
E. Membrane Damage Growth and Differentiation ○ DNA
F. Damage to DNA and A. Adaptation ○ ATP Production
Proteins B. Intracellular Accumulation ○ Cell membranes
IV. Reversible vs Irreversible C. Pathologic Calcifications ○ Protein synthesis
Injury D. Cellular Aging
A. Reversible Injury
LEGEND
Remember Lecturer Book Presentation

I. INTRODUCTION
● Pathology is the study of the structural, biochemical, and functional
changes in cells, tissues, and organs that underlie disease
○ General Pathology – Common reactions of cells and tissues to
injurious stimuli (e.g., acute inflammation)
○ Systemic Pathology – Alterations and underlying mechanisms in
diseases of particular organ systems.
● 4 aspects of a disease process that form the core of pathology:
○ Etiology – Initiating cause of the disease
■ Genetic (inherited or acquired mutations, disease associated
gene variants, or polymorphisms)
■ Environmental (infectious, nutritional, chemical, physical)
Multifactorial – Effect of various environmental insults on a
genetically susceptible individual
○ Pathogenesis – Sequence of molecular, biochemical, and
Figure 1. Stages of the Cellular Response to Stress and Injurious Stimuli
cellular events that lead to the development of the disease
○ Morphologic changes – Structural alteration in cells or tissues A. CAUSES OF CELL INJURY
that are characteristic of a disease; diagnostic of an etiological
● Hypoxia
process
○ Deficiency of oxygen
○ Clinical manifestations – End results of genetic, biochemical,
○ Causes cell injury by reducing aerobic oxidative respiration
and structural changes in cells and tissues are functional
○ Causes: reduced blood flow (ischemia),
abnormalities that lead to the clinical manifestations (signs and
○ inadequate oxygenation of blood due to cardiorespiratory failure,
symptoms) of disease, and its progression (clinical course and
decreased oxygen-carrying capacity of blood (i.e., anemia or
outcome)
carbon monoxide poisoning), severe blood loss
○ The most common cause of hypoxia is ischemia resulting from an
II. OVERVIEW OF CELLULAR RESPONSES TO arterial obstruction, but oxygen deficiency can also result from
STRESS AND NOXIOUS STIMULI inadequate blood oxygenation (due to the reduction in the oxygen
● Cellular Injury carrying capacity of the blood, anemia of any cause, or carbon
○ Occurs when cells are stressed so severely that the cells can no monoxide poisoning).
longer adapt. ● Toxins
○ Cells are exposed to inherently damaging agents or suffer from an ○ Potentially toxic agents such as air pollutants, insecticides, CO
intrinsic abnormality (carbon monoxide), asbestos, cigarette smoke, ethanol, and
○ The injury can sometimes be reversible within certain limits and drugs are among those that we encounter daily.
the cells can return to their stable baseline. ○ Even drugs in therapeutic doses can cause cell or tissue injury in
○ It can also be irreversible or can lead to cell death if the stress is a susceptible patient or in many individuals if these are used
severe, persistent, or rapid in onset. inappropriately or excessively.
● Cellular Adaptation ○ Innocuous substances such as glucose, salt, water, and oxygen
○ Our cells can change themselves in the face of stress. can be toxic.
○ Reversible; change in either the size of the cell and the number of ● Infection
the cell or the phenotype or function of the cell in the environment. ○ All types of disease-causing pathogens such as viruses, bacteria,
■ Hypertrophy – Increase in size and functional activity fungi, and protozoans injure cells
■ Atrophy – Decrease in size and metabolic activity ○ Submicroscopic viruses to tapeworms
■ Hyperplasia – Increase in cell number ○ Rickettsia, Bacteria, fungi, and higher forms of parasites
■ Metaplasia – Change in phenotype of the cells

1
● Immunologic Reactions cell death or in slowly progressive injury that becomes
○ Injurious reaction to endogenous self-antigens irreversible in time and eventually leads to cell death.
○ Reaction to viruses and environmental substances 2. The consequences of cell injury depend on the type, state, and
○ Excessive or chronic immune responses to microbes adaptability of the injured cell.
● Congenital / Genetics ○ The cell’s response to injury depends on its nutritional and
○ Enzyme defects in inborn errors of metabolism hormonal status, metabolic demands, and functions. When a
○ Accumulation of damaged DNA or misfolded proteins skeletal muscle cell of the leg has inadequate blood supply, it
○ Polymorphisms can be rested and preserved. Cardiac muscle cells, however,
● Physical Agents don’t have that option. With the complete mapping of the
○ Mechanical trauma human genome, there is great interest in identifying genetic
○ Extremes of temperature (burns and deep cold) polymorphisms that affect different individuals’ responses to
○ Sudden changes in atmospheric pressure various injurious agents.
○ Radiation 3. Any injurious stimulus may simultaneously trigger multiple
● Nutritional Imbalances interconnected mechanisms that damage cells.
○ Protein-calorie deficiencies ○ This explains why it can be difficult to ascribe cell injury in a
○ Vitamin deficiencies particular situation to a single or even dominant biochemical
○ Self-imposed nutritional shortage (anorexia nervosa) derangement.
○ Nutritional excess (obesity)
○ One of the major causes of cell injury include protein calorie A. ATP DEPLETION
insufficiency among impoverished populations. ● When the oxygen supplies decrease, the electron transport chain
● Chemical Agents and Drugs can’t work properly. Oxygen is needed as the final electron
○ Glucose or salt hypertonic concentrations acceptor.
○ Poisons (e.g., arsenic, cyanide, mercury) ● Decreased level of ATP causes failure of Na+K+ATPase to pump
○ Environmental pollutants, insecticides, ● Causes ribosomal detachment from rough endoplasmic reticulum
○ Carbon monoxide and asbestos due to swelling.
○ Alcohol and therapeutic drugs herbicides ● Also causes the following:
● Aging ○ Generalized swelling of the cell and its organelles
○ Aging of cells results in their diminished ability to respond to ○ Clumping of nuclear chromatin
stress and, eventually, death of the cells and of the organism. ○ Blebbing of the plasma membrane

B. PROGRESSION OF CELL INJURY AND DEATH B. MITOCHONDRIAL DAMAGE


● All stresses and noxious influences exert their effects first at the ● Cytosolic calcium, reactive oxygen species, and hypoxia can
molecular or biochemical level. damage the mitochondria.
● There is a time lag between the stress and the morphologic changes ● Causes the formation of a high conductance channel called a
of cell injury or death. The early changes are subtle and are only mitochondrial transition pore.
detected with highly sensitive examination methods. ○ The mitochondrial membrane becomes abnormally permeable
● With histochemical, ultrastructural, or biochemical techniques, these and membrane potential is lost and ATP production ceases.
morphologic changes may be seen in minutes to hours after injury.
Changes visible by light microscopy or the naked eye may take CONDUCTANCE
longer (hours to days) to appear. ● Increased conductance – more channels are open
● Morphologic manifestations of necrosis take more time to develop ● Decreased conductance – more channels are closed
than those of reversible damage. ● Causes: increase in cytosolic calcium, ROS, and hypoxia
○ In ischemia of the myocardium, cell swelling is a reversible
change that may take place in a matter of minutes and it is an
APOPTOSIS
indicator of ongoing cellular damage that may progress to
irreversibility within 1-2 hours ● The mitochondria sequester between their outer and inner
membranes several proteins that are capable of activating apoptotic
III. MECHANISM OF CELL INJURY pathways.
○ These include cytochrome C and proteins that indirectly activate
apoptosis-inducing enzymes called caspases.
Note: See APPENDIX A

C. INFLUX OF CALCIUM AND LOSS OF CALCIUM


HOMEOSTASIS
● Ca+ levels in the cytosol are extremely low, less than 0.1
micromoles compared to 1.3 millimoles outside the cell.
● “How do you keep the Calcium levels low in the cytosol? By
sequestering them into the mitochondria and into the ER, namely
the Sarcoplasmic Reticulum.”
● When there is ischemia or the cell is exposed to a toxin, the calcium
level increases inside the cell due to the release of the ions from the
mitochondria and from the ER.
● The calcium ions outside the cell will influx into the cell increasing
the concentration of cytosolic calcium and this will lead to
mitochondrial permeability and activation of enzymes like HPases,
phospholipases, proteases, and endonucleases.
○ Mitochondrial permeability is due to the opening of the
Figure 2. Effects of Myocardial Ischemia (diagram depicts what happens to cells mitochondrial permeability transition pore
when you don’t feed them enough oxygen)
○ Increased cytosolic calcium activates enzymes with potentially
harmful effects:
● The molecular alterations that result to cell injury are complex, but
■ Phospholipases – cause membrane damage
several principles are relevant to most forms of cell injury:
■ Proteases – break down both membrane and cytoskeletal
1. The cellular response to injurious stimuli depends on the nature of
proteins
the injury, its duration, and its severity.
■ Endonucleases – responsible for DNA and chromatin
○ Small doses of a chemical toxin or short periods of ischemia
fragmentation
may cause reversible injury, while large doses on the other
■ ATPases – hastens ATP depletion
hand and more prolonged ischemia can result either in rapid

2
● Elevated intracellular calcium levels also result in the induction of
apoptosis by direct activation of caspases and by increasing
mitochondrial permeability.

Figure 4. The generation, removal, and role of reactive oxygen species (ROS) in
cell injury.

● These causes free radicals to form:


1. Redox reactions of normal metabolism
2. Absorption of radiant energy
3. Rapid burst of ROS during inflammation
4. Metabolism of exogenous chemicals or drugs
5. Transition metals

E. MEMBRANE DAMAGE
Figure 3. The role of increased cytosolic calcium in cell injury. ● Common cause of membrane damage is hypoxia.
○ Oxygen deprivation leads to ATP depletion leads to decreased
D. ACCUMULATION OF OXYGEN-DERIVED FREE phospholipid synthesis then membrane damage.
RADICALS (OXIDATIVE STRESS) ● Too much Ca+ in the cytosol can also damage the cell membrane.
● These are oxygen derived free radicals. ○ When the cells die membrane integrity is lost and intracellular
● Free radicals are autocatalytic molecules. enzymes leak out and it can be measured in the blood.
● They react with molecules like carbohydrates or proteins and turn ○ Detection of these proteins in the circulation serves as a clinical
them into free radicals setting off a chain reaction. marker of cell death and organ injury. (Ex: CPK-MB, lactate
● “It is theorized that organisms age because cells accumulate free dehydrogenase: markers of myocardial injury; transaminases:
radicals over time.” hepatitis markers; amylase and lipases: pancreatitis markers;
● Examples of free radicals: alkaline phosphatase: biliary obstruction)
○ Superoxide anion – 1 electron ● Several biochemical mechanisms may contribute to membrane
○ Hydrogen peroxide – 2 electrons damage:
○ Hydroxyl radicals – 3 electrons ○ ROS
● Antioxidants are reducing agents that limit oxidative damage to ■ Oxygen free radicals cause injury to cell membranes by lipid
biological structures peroxidation.
○ (Ex: glutathione, vitamin A, vitamin E, vitamin C) ○ Decreased phospholipid synthesis.
■ Catalase – Present in peroxisomes; decomposes H2O2 ○ Increased phospholipid breakdown
■ Superoxide dismutases (SODs) – Converts superoxide anion ■ Due to activation of calcium-dependent phospholipases by
to H2O2 increased cytosolic and mitochondrial calcium, potentially
■ Glutathione peroxidase – Also catalyzes free radical resulting in changes in permeability and electrophysiologic
breakdown (H2O2 + reduced glutathione → Oxidized alterations of the membrane.
Glutathione + 2 H2O) ○ Cytoskeletal abnormalities
● Free reactive oxygen hurts cells by oxidizing lipids and disrupting ■ When cell swelling occurs (particularly in myocardial cells), this
cell membranes of organelles. damage results in the detachment of cell membrane from the
○ Can oxidize proteins rendering molecules useless and oxidize cytoskeleton, making it susceptible to stretching and rupture.
DNA so when mutated DNA breaks down, cell injury occurs.
● How do free reactive species injure the cells? Consequences of Membrane Damage
○ Lipid peroxidation → disruption of cell membranes. Can also 1. Mitochondrial membrane damage
oxidize proteins, especially enzymes. When an enzyme gets 2. Plasma membrane damage that leads to the loss of osmotic balance
oxidized, → lose activity → cell injury. Can also oxidize DNA → and influx of fluids and ions, as well as loss of cellular contents
breakdown → cell injury. 3. Injury to lysosomal membranes that result in leakage of their
● Removal of free radicals: enzymes into the cytoplasm and activation of acid hydrolases in the
○ Antioxidants (Vitamin A, C and E; Glutathione) acidic intracellular pH of the injured cell
○ Metals (Iron and Copper) binding to storage proteins (transferrin,
ferritin, lactoferrin and ceruloplasmin) that prevent the metals from
participating in ROS generating reactions.
○ Enzymes that act as free radical-scavenging systems
■ Catalase – Decomposes H2O2
■ Superoxidase dismutases – Converts O2- (superoxide anion)
to H2O2
■ Glutathione peroxidase – Catalyzes free radical breakdown

3
1. The inability to reverse mitochondrial dysfunction (lack of
oxidative phosphorylation and ATP generation) even after
resolution of the original injury
2. Profound disturbances in membrane function.
● Two types of Irreversible Injury:
1. Necrosis
2. Apoptosis

Note: See Appendix C

V. NECROSIS
● A pathologic process that is the consequence of severe injury.
● Necrosis-associated leakage of intracellular proteins through
damaged plasma membranes and ultimately into the circulation is
the basis for blood tests that detect tissue-specific cellular injury
● Spectrum of morphologic changes that follow cell death in a living
tissue due to progressive degradation action of enzymes on lethally
injured cells.
● Necrotic cells show increased eosinophils in H&E stains
Figure 5. Mechanisms of Membrane Damage in Cell Injury. ● What does necrosis look like under the microscope/Morphologic
features:
1. Increased eosinophilia in the cytoplasm
● Mechanisms of Membrane Damage in Cell Injury:
2. Bubbles or vacuoles inside the cell
○ Decreased oxygen and increased cytosolic Calcium are typically 3. Cytoplasm will have a glassy homogeneous appearance
seen in ischemia but may accompany other forms of cell injury. 4. Nuclear shrinkage
Reactive oxygen species, which are often produced on 5. Fragmentation and dissolution
reperfusion of ischemic tissues, also cause membrane damage 6. Breakdown of plasma membrane and organelle membrane
(not shown). 7. Abundant myelin figures
8. Leakage and enzymatic digestion of cellular contents
F. DAMAGE TO DNA AND PROTEINS
9. Nuclear changes:
● Chaperone proteins assist protein folding and save misfolded
■ Pyknosis (Condensation/shrinkage)
proteins and prevent protein degradation.
■ Karyorrhexis (Fragmentation)
● When the cell makes defective and useless proteins they go back to
■ Karyolysis (Dissolution)
ER for reprocessing or to proteasomes to be shredded.
○ Chaperone proteins prevent this from happening.
● Normally, after the proteins are synthesized in the ribosomes, they
will fold themselves into three dimensional architectural structures.
○ They have a quality control system in the form of chaperone
proteins.
○ They assist protein-folding and rescue misfolded proteins.
○ They also prevent protein degradation. They don’t send the cell to
proteasomes for destruction.
● Activates sensors that trigger p53-dependent pathways
○ Arrests cells in the G1 phase of the cell cycle and activates DNA
repair mechanisms.
○ If these mechanisms fail to correct DNA damage, p53 triggers
apoptosis by the mitochondrial pathway
Note: See APPENDIX B

IV. REVERSIBLE vs IRREVERSIBLE INJURY


A. REVERSIBLE INJURY
● Characterized by functional and structural alterations in early stages
or mild forms of injury, which are correctable if the damaging FIGURE 6. Necrotic Cardiac Myocytes
stimulus is removed
● Characteristics: A. COAGULATIVE NECROSIS
○ Cellular swelling – Results from influx of water
● A form of necrosis in which the architecture of dead tissue is
■ Earliest manifestation of almost all forms of injury to cells
preserved for a span of at least some days.
○ Fatty change – It results when toxic injury disrupts metabolic
● Structural proteins and enzymes are denatured but architectural
pathways and leads to rapid accumulation of triglyceride-filled
outline is still preserved.
lipid vacuoles
○ Plasma membrane blebbing and loss of microvilli
○ Mitochondrial swelling
○ Dilation of the ER
○ Accumulation of Myelin figures in cytosol
○ Eosinophilia (due to decreased cytoplasmic RNA)
● The injured cell goes back to normal once an injury stimulator agent
is removed.
● Hallmarks: cellular swelling, fatty change (examples: myocytes and
hepatocytes)
● Ultrastructural findings: Mitochondrial and ER swelling and bleb
formation

B. IRREVERSIBLE INJURY
● Happens if the injurious stimulus is persistent or severe the cell
suffers and ultimately undergoes cell death. FIGURE 7. Coagulative Necrosis (Kidneys)
● Cell does not go back to normal despite the
removal of the injury agent
● Two phenomena consistently characterize irreversibility:

4
● Wedge-shaped peel lesion is the infarct. The apex of the wedge
tends to point to the source of occlusion.
● Pale color tends to occur in solid organs with a single blood supply
such as the kidney, spleen, and the heart.
● Organs with dual blood supply such as the lung or the intestine tend
to be reddish and hemorrhagic in appearance.
● Ischemia caused by obstruction in a vessel may lead to coagulative
necrosis of the supplied tissue in all organs except the brain.
● Infarct is a localized area of coagulative necrosis
○ Organs without collateral circulation will have pale infarcts.

FIGURE 10. Brain in liquefactive necrosis

● Liquefactive necrosis due to infarction.

FIGURE 8. Section of the kidney under the microscope

● Viable part which is bluish in color and still intact and the necrotic
part shows fadedness.

B. LIQUEFACTIVE NECROSIS

FIGURE 11. Post recovery of brain liquefactive necrosis

● Brain lesions would appear cystic on CT scan, microbial cells and


circulating monocytes would remove much of the nuclear debris
leaving a cavity or cortical depression.
● There would be a peripheral gliosis and proliferation and
macrophages eating out the debris.

C. ENZYMATIC FAT NECROSIS


● Refers to focal areas of fat destruction.
● Typically resulting from release of activated pancreatic lipases into
the substance of the pancreas and the peritoneal cavity.
○ Pancreatic enzymes leak out of acinar cells and liquefy the
FIGURE 9. Liver abscess membranes of fat cells in the peritoneum.
○ The released lipases split the triglyceride esters contained within
● Characterized by digestion of the dead cells, resulting in fat cells.
transformation of the tissue into a viscous liquid. ● The fatty acids combine with calcium to produce grossly visible
● Area turned into mush. chalky-white areas (fat saponification)
● Wiped out due to the hydrolytic enzymes ○ This enables the surgeon and the pathologist to identify the
● Liquefaction by leukocyte enzymes is called superration and the lesions.
resultant fluid is pus (frequently creamy yellow) ○ Saponification- it is the same process of making commercialized
● Seen in focal bacteria/fungal infections soap (chalky appearance)
● Cellular injury leads to release of powerful enzymes which damage
fat by production of calcium soaps.
● Damage to the fat cells releases triacylglycerides.
○ The lipases break down the triglycerides into fatty acids which are
negatively charged molecules that attract positively charged
calcium. Calcium Ions forming these soaps.

5
Figure 12. Gross examination of a Pancreas

● Notice the white-chalky areas where saponification occurred.


Figure 14. Tuberculosis of the lung, with a large area of caseous necrosis

● On microscopic examination, the necrotic area appears as a


structureless collection of fragmented or lysed cells and amorphous
granular debris enclosed within a distinctive inflammatory border
○ This appearance is characteristic of a focus of inflammation
known as a granuloma.

Figure 13. Fat Necrosis under the microscope

● Necrotic cells show vague outlines, loss of nuclei and their


cytoplasm.
● There are some viable steatosis at the left which are not necrotic.
Chronic fat cells on the right have vague cellular outlines that have
lost their peripheral nuclei and their cytoplasm has become pink.
● Amorphous mass of necrotic material is another microscopic tip.
Look for unhealthy fat cells with deposition of calcium.

D. TRAUMATIC FAT NECROSIS Figure 15. Higher magnification of the Granuloma from TB
● A lump of dead or damaged breast tissue that sometimes appears
after breast surgery, radiation or another trauma. ● Granulomas are nodular lesions made up of modified macrophages
○ Common scenario: “Kunwari athlete yung babae tapos nagkaroon called epithelioid cells.
siya ng breast mass, or yung babae may history ng domestic ● Systemic symptoms of tuberculosis are a consequence of cytokines
abuse at nagkaroon siya ng breast mass, this is what we call as like tumor necrosis factor and interleukin-1 are released from the
activated macrophages.
traumatic fat necrosis”.

E. CASEOUS NECROSIS F. FIBRINOID NECROSIS


● There is a yellowish cheese-like lesion in the hilum of this lung (Left)

● Yellowish necrosis as a combination of coagulation and


liquefactive necrosis.
○ This is caseous necrosis due to tuberculosis.
● Encountered most often in foci of tuberculous Infection
○ Not all caseous necrosis is caused by tuberculosis
● The term caseous (cheese-like) is derived from the friable white
appearance of the area of necrosis

Figure 16. Fibrinoid Necrosis seen in vasculitis

6
● Necrotic tissue histologically resembles fibrin. The lesion has a big Table 1. Examples of apoptosis in physiological conditions
homogeneous appearance.
● Usually seen in immune reactions involving blood vessels The removal of Critical for involution of
supernumerary cells (in primordial structures and
● Occurs when complexes of antigens and antibodies are deposited in excess of the required remodelling of maturing tissues
the walls of arteries number) during development
● Result in a bright pink and amorphous appearance in H&E stains
called “fibrinoid” (fibrin-like)
Involution of hormone- Endometrial cell breakdown
● Immunologically mediated vasculitis syndromes dependent tissues on during the menstrual cycle,
hormone withdrawal ovarian follicular atresia in
G. GANGRENOUS NECROSIS menopause, and regression of
the lactating breast after
weaning

Cell turnover in proliferating Immature lymphocytes in the


cell populations bone marrow and thymus, B
lymphocytes in germinal centers
that fail to express useful
antigen receptors, and epithelial
cells in intestinal crypts, to
maintain a constant cell number

Elimination of potentially Prevent immune reactions


harmful self-reactive against one’s own tissues
lymphocytes

Death of host cells that have Neutrophils in an acute


served their useful purpose inflammatory response, and
lymphocytes at the end of an
Figure 17. Gangrenous Necrosis
immune response
● Gross term to describe dead tissue.
● Apoptosis in Pathologic Conditions
● Common sites include lower limbs, gallbladder, gastrointestinal
○ Eliminates cells that are injured beyond repair
tract.
○ Limiting collateral tissue damage
● Microscopically, gangrenous necrosis can be coagulative or
liquefactive. Table 2. Examples of Apoptosis in pathologic conditions
○ If it is coagulative, it is called dry gangrene
○ If it is liquefactive it is called wet gangrene DNA damage Radiation and cytotoxic
● Not a specific pattern of cell death anticancer drugs can damage
DNA
● Applied to a limb, generally the lower leg that has lost its blood
supply and has undergone necrosis (typically coagulative necrosis
Accumulation of misfolded Cell death triggered by
● Bacterial infection is more of liquefactive necrosis aka wet gangrene
proteins improperly folded intracellular
Note: See Appendix D proteins and the subsequent
endoplasmic reticulum (ER)
stress response
VI. APOPTOSIS
● Apoptosis is a pathway of cell death in which cells activate enzymes
that degrade its own nuclear DNA and nuclear and cytoplasmic Infections An important host response to
proteins. viruses consists of cytotoxic T
● Divided into the initiation phase and the execution phase. lymphocytes which induce
● Induced by a tightly regulated suicide program in which cells apoptosis of infected cells in an
destined to die activate intrinsic enzymes that degrade the cells’ attempt to eliminate reservoirs
genomic DNA and nuclear and cytoplasmic proteins of infection.
● Break up into plasma membrane–bound fragments, called apoptotic
bodies Pathologic atrophy in Such as occurs in the pancreas,
● While the plasma membrane remains intact, its surface components parenchymal organs after parotid gland, and kidney.
are altered so as to produce “find me” and “eat me” signals for duct obstruction
phagocytes
● Dead cell and its fragments are rapidly devoured, before the
contents leak out B. MECHANISMS OF APOPTOSIS
● Apoptosis does not elicit an inflammatory reaction ● Results from the activation of enzymes called “Caspases”
● Initiation Phase ● Caspases
○ For activation of enzymes ○ Proteases containing a cysteine in their active site and cleave
● Execution Phase proteins after aspartic residues
○ Cells are dying ○ Caspases exist as inactive proenzymes and must undergo
enzymatic cleavage to become active
A. CAUSES OF APOPTOSIS ○ Presence of active caspases is therefore a marker for cells
● Apoptosis in Physiological Situations undergoing apoptosis
○ It serves to eliminate cells that are no longer needed ● Process of apoptosis is divided to:
○ Humans turn over almost 1 million cells per second ○ Initiation phase - Some caspases become catalytically active
○ Cells undergo apoptosis because they are deprived of necessary and unleash a cascade of other caspases
survival signals ○ Execution phase - Terminal caspases trigger cellular
○ They receive pro-apoptotic signals from other cells or the fragmentation
surrounding environment ● 2 distinct pathway on caspase activation
○ Mitochondrial pathway
■ Responsible for apoptosis in most physiologic and pathologic
situations

7
■ From increased permeability of the mitochondrial outer
membrane with consequent release of death-inducing
(pro-apoptotic) molecules from the mitochondrial
intermembrane space into the cytoplasm
■ Release of pro-apoptotic proteins such as cytochrome c
■ Anti-apoptotic – BCL2, BCL-XL, and MCL1
■ Pro-apoptotic – BAX and BAK
■ Regulated apoptosis initiators – BAD, BIM, BID, Puma, and
Noxa
○ The Extrinsic (Death Receptor-Initiated) Pathway
■ Initiated by engagement of plasma membrane death receptors
■ Death receptors - Members of the tumor necrosis factor
(TNF) receptor family and contains a cytoplasmic domain involved
in protein-protein interactions
■ Death domain - Essential for delivering apoptotic signals
■ Type 1 TNF receptor (TNFR1) and a related protein called Fas
(CD95) - Best known death receptors
■ FasL is expressed on T cells that recognize self antigens (and
functions to eliminate self-reactive lymphocytes that also
express the receptor Fas upon recognition of self antigens)
■ When FasL binds to Fas, three or more molecules of Fas are
brought together, and their cytoplasmic death domains form a
binding site for an adaptor protein called FADD (Fas-associated
death domain).
■ Once attached to this complex, FADD binds inactive caspase-8 Figure 19. The Intrinsic Pathway of Apoptosis. (Does not need a death receptor)
(or caspase-10), bringing together multiple caspase molecules
and leading to autocatalytic cleavage and generation of active
caspase-8.
■ Active caspase-8 initiates the same executioner caspase
sequence as in the mitochondrial pathway.
■ Extrinsic apoptosis pathway can be inhibited by a protein called
FLIP, which binds to procaspase-8, thereby blocking
■ FADD binding, but cannot activate the caspase.

Figure 20. Apoptosis in Viral Hepatitis (arrow)

● Example of hepatocytes that are dying individually as indicated by


the arrows. From injury through an infection by viral hepatitis.

C. OTHER PATHWAYS OF CELL DEATH


NECROPTOSIS
● Initiated by engagement of TNF receptors.
● The name necroptosis implies that there are features of both
necrosis and apoptosis.
● Hypothesized to play a role in ischemic injury and those associated
with inflammatory reactions in which the cytokine TNF is produced.
● The signals leading to necroptosis do not result in caspase
activation, and hence it is also sometimes referred to as
“caspase-independent” programmed cell death.
● Process of necroptosis starts in a manner similar to that of the
extrinsic form of apoptosis.
● Necroptosis involves two kinases called receptor-interacting protein
kinase 1 and 3 (RIPK1 and RIPK3).
● Physiologic necroptosis occurs during the formation of the
Figure 18. The Extrinsic Pathway of Apoptosis mammalian bone growth plate.
● It is associated with cell death in steatohepatitis, acute pancreatitis,
● The death receptors represented in the diagram are members of the ischemia-reperfusion injury, and neurodegenerative diseases such
TNF family and mutate passively. as Parkinson disease.
● The fas ligand binds to the fas receptor cross linking of the three or
more fas receptors and creates a binding site for an adapter PYROPTOSIS
molecule called FADD (Fas Associated Death Domain)
● Associated with activation of a cytosolic dangersensing protein
complex called the inflammasome.
● The name pyroptosis stems from the association of apoptosis with
fever (Greek, pyro = fire).
● Form of apoptosis that is accompanied by the release of the
fever-inducing cytokine IL-1.

8
● Microbial products that enter infected cells are recognized by PATHOLOGIC AND PHYSIOLOGIC HYPERTROPHY
cytoplasmic innate immune receptors and can activate the
multiprotein complex called the inflammasome. PATHOLOGIC PHYSIOLOGIC

FERROPTOSIS ● The striated muscles in the ● Enlargement of uterus during


heart and skeletal muscles pregnancy that results from
● Triggered when excessive intracellular levels of iron or reactive
have only a limited capacity hypertrophy of smooth
oxygen species overwhelm the glutathione-dependent antioxidant
for division and respond to muscle fibers (due to
defenses to cause unchecked membrane lipid peroxidation.
increased metabolic demands Estrogen influence)
● Widespread peroxidation of lipids disrupts many aspects of
mainly by undergoing ● Increased muscle mass of
membrane function, including fluidity, lipid-protein interactions, ion
hypertrophy body builders.
and nutrient transport, and signaling pathways.
● Most common stimulus for
● Overall effect is the loss of plasma membrane permeability, which
hypertrophy is increased
ultimately leads to cell death resembling necrosis.
workload.
● Classic Example: Heart
VII. AUTOPHAGY enlargement in response to
● A process in which a cell eats its own contents. pressure overload.
● It involves the delivery of cytoplasmic materials to the lysosome for
degradation.
● An evolutionarily conserved survival mechanism in states of nutrient ● Due to the synthesis and assembly of additional intracellular
deprivation, starved cells live by cannibalizing themselves and structural components
recycling the digested contents. ● Mechanism of Hypertrophy:
● Prominent in atrophic cells exposed to severe nutrient deprivation. 1. Mechanical sensors in the cell detects the increased load
● Can trigger cell death if it is inadequate to cope with the stressor. 2. These sensors activate the signaling pathways such as PI3K/AKT
● Involved in the turnover of organelles like the ER, mitochondria, and (physiologic e.g. exercise induced hypertrophy) & GPCR
lysosomes and the clearance of intracellular aggregates that (pathologic)
accumulate during aging, stress, and various disease states. 3. It also stimulates the production of growth factors (TGFB, IGF-1,
Fibroblast growth
STEPS IN AUTOPHAGY 4. They activate the transcription factors including: GATA4, NFAT
(nuclear factor of activated T cells) & myocyte enhancer factor 2
1. Nucleation and formation of an isolation membrane called
(MEF2).
phagophore.
2. Formation of a vesicle, called the autophagosome, from the isolation
membrane, inside which intracellular organelles.
3. Maturation of the autophagosome by fusion with lysosomes, to
deliver digestive enzymes that degrade the contents of the
autophagosome.
○ Autophagosome formation is regulated by more than a dozen
proteins that act in a coordinated and sequential manner.

Note: See Appendix E

VIII. ADAPTATION OF CELLULAR GROWTH AND


DIFFERENTIATION
A. ADAPTATION
● Reversible changes in the size, number, phenotype, metabolic
activity, or functions of cells in response to changes in their
environment.

Figure 22. Biochemical Mechanisms of Myocardial Hypertrophy. The major


known signaling pathways and their functional effects are shown.

Figure 21. Cellular Adaptation

HYPERTROPHY
● An increase in the size of cells and of the affected organ
● A result of increased cellular protein production
● Hypertrophy can be physiologic or pathologic
Figure 23. Cardiac Hypertrophy (left ventricle)
● Hypertrophied organ has no new cells
● The heart weight varies with the body height and weight. It averages
approximately 250-300 grams in females and 300-350 grams in

9
males.The usual thickness of the free wall of the right ventricle is ● There is thickening of the endometrium.
0.3-0.5 cms and on the left ventricle 1.3-1.5 cms. ○ This is physiologic because of exposure to estrogen.
● Increases in cardiac size and weight accompany many forms of ○ This was removed to rule out systemic hyperplasia.
heart disease
● This is Pathologic Hypertrophy because the process is maladaptive ATROPHY
overtime. ● A reduction in the size of an organ or tissue due to a decrease in cell
● In contrast, athletic training leads to minimal increase in your heart size and number.
size but marked increase in your cardiac function. ● Cerebral atrophy seen here is mainly on the frontal and parietal
● This heart has undergone cardiac hypertrophy involving the left regions as characterized by narrow gyri along with widened sulci.
ventricle. ● Alzheimer ’s disease is marked by progressive loss of higher mental
● The number of myocardial fibers doesn't increase but its size functions. The neocortical neuronal loss and the neuritic plaque is
increase in response to increased workload primarily responsible for the loss of higher mental functions.
● This is systemic hypertension leading to thickening of your left
ventricle. PATHOLOGIC AND PHYSIOLOGIC ATROPHY
○ This is what you called compensatory hypertrophy; it is pathologic
because this can get maladaptive overtime. PATHOLOGIC PHYSIOLOGIC
○ There will be eventual myocyte loss, necrosis and ventricular
dysfunction. ● Notochord and thyroglossal ● Decreased workload
○ This is a left sided heart failure that can lead to congestion. duct undergo atrophy during ● Loss of innervation
○ Case Sample: Patient has a history of chronic renal failure and fetal development (denervation atrophy)
heavy heart. ● The decrease in the size of ● Diminished blood supply
■ Because your kidneys also play a role in blood pressure uterus after parturition ● Inadequate nutrition
control. ● Loss of endocrine stimulation
■ Kidney failure leads to secondary hypertension. ● Pressure (Tissue
■ Primary hypertension usually we don't know the cause, compression)
idiopathic.

HYPERPLASIA
● Increase in the number of cells in an organ or tissue in response to a
stimulus
● Can only take place if the tissue contains cells capable of dividing.
● Result of growth factor–driven proliferation of mature cells and, in
some cases, by increased output of new cells from tissue stem cells

PATHOLOGIC AND PHYSIOLOGIC HYPERPLASIA


PATHOLOGIC PHYSIOLOGIC

● Caused by excessive or ● The action of hormones or


inappropriate actions of growth factors occur when
hormones or growth factors there is a need to increase
acting on target cells. functional capacity of
○ Endometrial hyperplasia hormone sensitive organs, or
○ Benign prostatic when there is need for
hyperplasia. compensatory increase
damage or resection. Figure 25. Brain of a Person with Alzheimer’s Disease
○ Proliferation of glandular
epithelium of female breast ● Mechanism of Atrophy
at puberty ○ Atrophy results from decreased protein synthesis and increased
○ Liver transplantation protein degradation in cells.
○ Protein synthesis decreases because of reduced trophic signals.
○ Degradation of cellular proteins occurs mainly by the
ubiquitin-proteasome pathway.
○ Atrophy is also accompanied by increased autophagy.
○ It is atrophic, has widened sulci and narrow gyri.
■ This happens in dementia which occurs in Alzheimer's
■ The cortical cells could be lost along with intraneuritic blocks
which is responsible for the loss of higher mental function.
■ Alzheimer’s disease is encoded in chromosome 21.

METAPLASIA
● Reversible change in which one differentiated cell type (epithelial or
mesenchymal) is replaced by another cell type
● It often represents the adaptive response in which one cell type that
is sensitive to a particular stress is replaced by another cell that is
better able to withstand the adverse environment.
● The most common epithelial metaplasia is columnar to squamous.
The examples are:
○ In the habitual cigarette smoker, the normal ciliated columnar
epithelial cells of trachea and bronchi are often replaced by
Figure 24. Total Hysterectomy of Uterus stratified squamous epithelial cells
○ Vitamin A deficiency can also induce squamous metaplasia in the
● Mechanisms of Hyperplasia respiratory epithelium
○ Hyperplasia is the result of growth factor-driven proliferation of ○ Stones in excretory ducts of salivary glands, pancreas or bile
mature cells, and in some cases, by increased output of new cells ducts which are normally lined by secretory columnar epithelium
from tissue stem cells. into squamous metaplasia.
○ Example of hyperplasia in cells forming the endometrial lining and
stroma

10
● Connective Tissue Metaplasia
○ Formation of cartilage, bone or adipose cells (mesenchymal
tissues) in tissues that normally do not contain these elements
■ For example, bone formation in muscle, designated myositis
ossificans that occasionally occurs after intramuscular
hemorrhage
○ However, this type of metaplasia is not associated with increased
cancer risk.
● Mechanisms of Metaplasia
○ Does not result from a change in the phenotype of an already
differentiated cell type
○ Rather, it results from either reprogramming of local tissue stem
cells, or alternatively, colonization by differentiation cell
populations from adjacent sites

Figure 28. Dysplasia in Uterine Cervix

● Ectocervix - non-keratinizing squamous epithelium


● Endocervix - tall columnar epithelium
○ The figure above is an example of cellular dysplasia. The normal
cervical squamous epithelium has transformed into a more
disorderly pattern growth of dysplastic epithelium.

B. INTRACELLULAR ACCUMULATION
● Four main mechanisms leading to abnormal intracellular
accumulations:
1. Inadequate removal of a normal substance secondary to defects
in packaging and transport, as in fatty change (steatosis) in the
liver.
Figure 26. Lining of the Larynx of a Chronic Smoker
2. Accumulation of an endogenous substance as a result of genetic
or acquired defects in its folding, packaging, transport, or
● It is metaplasia of laryngeal respiratory epithelium which can occur secretion, as with certain mutated forms of α1-antitrypsin.
in a smoker. 3. Failure to degrade a metabolite due to inherited enzyme
● The normal respiratory epithelium is pseudostratified then it deficiencies, typically lysosomal enzymes. The resulting disorders
becomes squamous when the lining gets irritated. are called lysosomal storage diseases.
● This is what you called squamous metaplasia 4. Deposition and accumulation of an abnormal exogenous
substance when the cell has neither the enzymatic machinery to
degrade the substance nor the ability to transport it to other sites.
● Lipids
○ Three major classes of lipids
1. Triglycerides
2. Cholesterol/ cholesterol esters
3. Phospholipids
● Steatosis (Fatty change)
○ Describe abnormal accumulations of triglycerides within
parenchymal cells.
○ Fatty change is often seen in the liver because it is the major
organ involved in fat metabolism, but it also occurs in the heart,
muscle, and kidney.
○ The causes of steatosis include toxins, protein malnutrition,
diabetes mellitus, obesity, and anoxia.

Figure 27. Intestinal Lining of the Esophagus

● This is intestinal metaplasia because the squamous lining of the


esophagus is converted into columnar type with goblet cells.
○ It is due to reflux of acid.
○ The esophageal sphincter is usually closed, but with reflux it is
always so there are changes for it to become more susceptible to
the acidic environment.
○ To protect the lining, it transforms to another type of cell with
plenty of goblet cells to produce mucous that can protect it from
the harmful acid
● The classic scenario for this is Barrett’s Esophagus or a patient who
has chronic gastroesophageal reflux (GERD).
● Metaplasia can be helpful at some point pero kung di nawawala
yung injurious stimulus, it will undergo a specific change.
Figure 29. Fatty Liver

11
● Cholesterol and Cholesteryl esters
○ Accumulations manifested histologically by intracellular vacuoles
are seen in several pathologic processes such as:
1. Atherosclerosis - smooth muscle cells and macrophages
within the intimal layer of the aorta and large arteries are filled
with lipid vacuoles
2. Xanthomas – Intracellular accumulation of macrophages
3. Cholesterolosis – focal accumulations of cholesterol-laden
macrophages in the gallbladder
4. Niemann-Pick disease, type C - lysosomal storage disease
caused by mutations with enzymes involved in cholesterol
trafficking
● Proteins
○ Intracellular accumulations of proteins usually appear as rounded,
eosinophilic droplets, vacuoles, or aggregates in the cytoplasm
○ Excesses of proteins within the cells sufficient to cause
morphologically visible accumulation have diverse causes such
as:
■ Reabsorption droplets in proximal renal tubules are seen in
renal diseases associated with protein loss in the urine
(proteinuria)
■ The ER becomes hugely distended, producing large, Figure 30. The Liver Cells and Lipofuscin
homogeneous eosinophilic inclusions called Russell bodies.
■ Defective intracellular transport and secretion of critical proteins ● These pigments may accumulate overtime as manifestation of old
■ Accumulation of cytoskeletal proteins age or can be clinically relevant.
■ Aggregation of abnormal proteins ● This is the liver cell and the pigments are lipofuscin.
● Hyaline change ● This accumulates overtime in the cells, particularly in the liver and
○ Usually refers to an alteration within cells or in the extracellular the heart as a result of aging.
space that gives a homogeneous, glassy, pink appearance in ● There is no clinical consequence.
routine histologic sections stained with H&E. ● This demonstrates the process of autophagocytosis.
○ It is widely used as a descriptive histologic term rather than a
specific marker for cell injury.
■ Intracellular hyaline accumulations of protein include
reabsorption droplets, Russell bodies, and alcoholic hyaline
■ Extracellular hyaline is more difficult to analyze
■ Collagenous fibers in old scars may appear hyalinized, but
change in biochemical basis is not clear
● Glycogen
○ Excessive intracellular deposits of glycogen are seen in patients
with an abnormality in either glucose or glycogen metabolism.
○ Glycogen masses appear as clear vacuoles within the cytoplasm
because glycogen dissolves in aqueous fixatives
■ It is most readily identified when tissues are fixed in absolute
alcohol
○ Diabetes mellitus
■ Prime example of glycogen metabolism disorder
■ Glycogen is found in renal tubular epithelial cells, as well as
within liver cells, beta cells of the islets of Langerhans within
the pancreas, and heart muscle cells
● Pigments
○ Exogenous Pigments
■ Carbon (coal dust), a ubiquitous air pollutant in urban areas – Figure 31. Lung Section and the Eosinophilic Pigments
most common. ● The brown coarsely granular material in macrophages/ histiocytes in
■ Picked up by macrophages within the alveoli, when inhaled, this alveolus is hemosiderin that has accumulated and as a result of
then transported through lymphatic channels to lymph nodes in breakdown of RBCs and release of Iron in heme. The Macrophages
the tracheobronchial region clear up this debris which is eventually recycled.
■ Anthracosis - accumulation of black pigment in lung tissues and ● A lung section and the brown pigments are eosinophilic that has
lymph nodes accumulated with the release of iron.
■ Coal Worker’s Pneumoconiosis - carbon dust may induce a ● The cells are macrophages and lymphocytes.
fibroblastic reaction or emphysema ● Macrophages are also called heart failure cells.
■ Tattooing - localized exogenous pigment of the skin; The ○ They act as garbage collectors; they will eat every debris in our
pigments inoculated are phagocytosed by dermal macrophages body.
○ Endogenous Pigments
■ Lipofuscin (lipochrome or wear-and-tear pigment) is an
insoluble pigment; Its importance lies in its being a telltale sign
of free radical injury and lipid peroxidation
■ Melanin is the only endogenous brown-black pigment; pigment
formed when the enzyme tyrosinase catalyzes the oxidation of
tyrosine to dihydroxyphenylalanine in melanocytes
■ Hemosiderin, a hemoglobin-derived, golden yellow to brown,
granular, or crystalline pigment is one of the major storage
forms of iron

12
C. PATHOLOGIC CALCIFICATIONS
● Abnormal tissue deposition of calcium salts, together with smaller
amounts of iron, magnesium, and other mineral salts
● 2 forms
1. Dystrophic calcification
■ Occurs locally in dying tissues
■ Occurs despite normal serum levels of calcium and in the
absence of derangements in calcium metabolism
2. Metastatic calcification
■ Occurs in normal tissues
■ Almost always results from hypercalcemia

Figure 32. Microscopic View of the Liver Stained with Prussian Blue

● Prussian blue reaction is seen with the iron stain of the liver which
demonstrates large amounts of hemosiderin that are present within
the cytoplasm of hepatocytes and Kupffer cells.

Figure 35. Dystrophic Calcifications


● Calcium deposition in abnormal tissues secondary to injury or
necrosis.
● Tends to be localized. Usually seen in Tuberculosis.
● Encountered in necrotic area
● Whether they are of coagulative, caseous, or liquefactive type, and
in foci of enzymatic necrosis of fat
● Almost always present in the atheromas of advanced
atherosclerosis
● Commonly develops in aging or damaged heart valves
● Calcium salts appear macroscopically as fine, white granules or
clumps, often felt as gritty deposits
● Sometimes a tuberculous lymph node is virtually converted to stone.
● Often a cause of organ dysfunction
● Serum calcium is normal in dystrophic calcification.

Figure 33. Surface of a Lung with Anthracotic Pigments

● Carbon particles have been ingested by pulmonary macrophages


and they are carried on along with the lymphatics through
interlobular septa of the pleural surfaces of the lung and eventually
in the hilar lymph nodes.
● These black pigments are anthracotic pigments.
● Usually you’ll find these pigments in smokers or people who have
been exposed in highly industrialized areas.

Figure 36. Dystrophic Calcification of Aortic Valve

● Usually occurs in bicuspid aortic valves.


● Sometimes in older individuals, a normal tricuspid valve will undergo
calcification the so-called senile calcific aortic stenosis.
● Nodules of calcifications are seen on the cusps here. These
calcifications tend to be localized and can be seen in lots of lesions.
● Sometimes it can form calcification in the liquefactive necrotic area,
abscess, fat necrosis, thrombi, parasites (Schistosoma), in certain
tumors and lesions.
● Dystrophic is not secondary to hypercalcemia.

Figure 34. Section of a Lung with Anthracotic Pigments

● There is no clinical consequence for anthracosis. However, in coal


miner's pneumoconiosis, you will have massive amounts of carbon
pigments leading to a restrictive lung disease.

13
REFERENCES
● Kumar, V., Abbas, A. K., Aster, J. C., & Perkins, J. A. (2018). Robbins Basic
Pathology (Tenth edition.). Philadelphia: Elsevier.
● 2024. 1.02 Cellular Responses to Stress: Adaptation, Injury and Death

Figure 37. Metastatic Calcification

● May occur in normal tissues whenever there is hypercalcemia.


● Has nothing to do with malignancy. This is a widespread deposition
of calcium in normal tissue and it is usually secondary to
hypercalcemia.
● Four principal causes of hypercalcemia:
1. Increased secretion of parathyroid hormone (PTH) with
subsequent bone resorption
2. Resorption of bone tissue, secondary to primary tumors of bone
marrow (e.g., multiple myeloma, leukemia) diffuse skeletal
metastasis (e.g., breast cancer), accelerated bone turnover (e.g.,
Paget disease), immobilization.
3. Vitamin D related disorders
4. Renal failure, which causes retention of phosphate, leading to
secondary hyperparathyroidism
● May occur widely throughout the body but principally affects the
interstitial tissues of the gastric mucosa, kidneys, lungs, systemic
arteries, and pulmonary veins
● Mineral salts cause no clinical dysfunction, but on occasion massive
involvement of the lungs produces remarkable x-ray images and
respiratory compromise.
● Massive deposits in the kidney (nephrocalcinosis) may in time cause
renal damage.

D. CELLULAR AGING
● The result of a progressive decline in cellular function and viability
caused by genetic abnormalities and the accumulation of cellular
and molecular damage due to the effects of exposure to exogenous
influences.
● Mechanisms that counteract cellular aging:
1. Accumulation of DNA Damage
■ Defective DNA repair mechanisms; conversely, caloric
restriction may activate DNA repair and slow aging in model
organisms
2. Replicative Senescence
■ Reduced capacity of cells to divide secondary to progressive
shortening of chromosomal ends (telomeres)
3. Defensive Protein Hemostasis
■ Resulting from impaired chaperone and proteasome functions
4. Nutrient Sensing System
■ Caloric restriction increases longevity. Mediators may be
reduced IGF-1 signaling and increased sirtuins

Note: See Appendix F

14
APPENDIX

.
APPENDIX A. Role of mitochondria in cell injury and death

APPENDIX B. Mechanisms of protein folding and the unfolded protein response.

15
APPENDIX C. Cell Injury and Cell Death

APPENDIX D. Types of Necrosis

16
APPENDIX E. Process of Autophagy

APPENDIX F. Mechanisms that Cause and Counteract Cellular Aging

17
A pharmacologist wants to investigate the effects of a novel drug Oxidized glutathione
on cultured liver cells. The drug is found to kill liver cells, Superoxide
unfortunately, and it seems to do this predominantly via Hydroxyl radical
activation of endonucleases that damage the nucleus. Which of
the following mechanisms of cell injury is the most direct in 8 A researcher keeps hepatocytes at zero degrees Celsius and
activating the endonucleases? treats them with trypsin to digest the receptor on the cell surface.
Calcium influx into the cell He then raised the temperature to 37 degrees Celsius. He adds
DNA damage leading to activation of proapoptotic proteins radioactive LDL to the culture media. Several hours later, he
Mitochondrial damage leading to increased production of found the labeled LDL to be inside the cell. This specific process
reactive oxygen species of LDL uptake differs from phagocytosis of damaged cells in
Breakdown of lysosomal membranes which of the following?
Mitochondrial damage resulting in decreased production of ATP Use of clathrin-coated pits
Use of membrane-enclosed vesicles in the uptake process
2 A surgical patient is administered a muscarinic agonist to Dependence on acidification
relieve his urinary retention. The diagram below depicts the Coupling with the lysosomal system
mechanism of action of the agent. What is the function of Use of hydrolases
molecule B?
9 Which one of the following structures would be most directly
Directly activates protein kinase C affected by a mushroom toxin that binds to actin subunits and
Binds to the endoplasmic reticulum disrupts actin function?
Hydrolysis of PIP2 to form DAG and IP3 Keratin fibers
Stimulation of Gi activity through phosphorylation of PIP2 Microtubules
Stimulation of Gs activity through phosphorylation of PIP2 Collagen fibers
Intermediate filaments
3 A lymph node removed from a 60-year-old man is diagnosed Microfilaments
as lymphoma. Cytogenetic analysis of the lymphoma reveals
translocation t(14;18). What is the role of the protein that is 10 A necropsy report describes a 5 x 3-cm focus of liquefactive
overexpressed as a result of this chromosomal rearrangement? necrosis. An occluded blood vessel is found to be the cause of
Antagonist of apoptotic cell death this lesion. In which of the following sections of the report would
Upregulation of the cell cycle this lesion most likely be described?
Constitutive activation of RAS Respiratory system
Cell cycle inhibitor in response to the detection of a DNA error Hepatobiliary system
Inactivation of p53 Central nervous system
Genitourinary system
4 A 9-year-old boy presents with a history of multiple bacterial Cardiovascular system
infections. Further workup reveals faulty phagolysosome
formation as the underlying problem. Which of the following 11 Which of the following cellular proteins is associated with the
structures is most likely defective in this boy’s cells? nuclear membrane?
Microtubules Actin
Microfilaments Lamin
Intermediate filaments Cytokeratin
Golgi apparatus Desmin
Proteosome Myosin

5 A 70-year-old woman has had a blood pressure of 160/100 12 A researcher writes a review article regarding prostate
mm Hg for many years. Abdominal ultrasound shows a carcinoma. He found that Rb, p53, and bcl-2 genes are involved
decrease in the size of the kidney. Which of the following in the development of prostate carcinoma. Which of the following
organelles plays a major role in the process that occurred in the mechanisms may be involved in the loss of cell cycle
affected kidney? control that occurs in prostate carcinoma?
Lysosome Increased CdkI activity
Endoplasmic reticulum Dephosphorylation of Rb
Mitochondria Decreased transcription of G1/S cyclin
Ribosome Increased transcription of gene regulatory proteins such as E2F
Golgi apparatus Decreased expression of bcl-2

6 Preparation for genome separation occurs in which of the 13 Metaplasia is the most likely cellular change exhibited in
following phases of the cell cycle? which of the following?
M The segmental bronchus of a 47-year-old man who smokes
G0 cigarettes two packs per day for 30 years
G1 The brain of an 85-year-old man who succumbed to
S complications of Alzheimer disease
G2 The liver of a patient who underwent partial hepatectomy two
weeks ago
7 A study subjects cultured cells to certain types of radiation. The breast of a 19-year-old woman who just gave birth to her
Subsequently, the intracellular proteins are evaluated for first child
evidence of cross-linking. Which of the following has the The toe of a 60-year-old diabetic woman with no palpable
greatest propensity for causing such protein changes? dorsalis pedis and posterior tibial pulses
Nitric oxide
Hydrogen peroxide
14 During a forensic autopsy, skin sections sampled from a mRNA
cadaver reveals dermal collections of histiocytes containing a siRNA
stippled, or somewhat chunky-appearing, yellow-brown miRNA
pigment. Prussian blue stain is done on the tissue sections lncRNA
which causes the pigments to appear bluish. Which of the tRNA
following is the most probable cause for the pigment?
Normal melanin accumulation in a darkly pigmented individual 20 A 9-month-old baby is brought to her pediatrician because
Abnormal protein accumulation in an alcoholic she has suffered recurrent respiratory tract infections. At birth,
Previous trauma that resulted in hemorrhage she had coarse facial features and musculoskeletal deformities.
Wear-and-tear pigment in an older individual Microscopic evaluation of her cells shows large inclusion bodies.
Cigarette smoking An enzyme deficiency in which of the following organelles is
responsible for her condition?
15 A 60-year-old man with chronic knee pain is diagnosed with Peroxisome
degenerative joint disease. The disease is characterized by loss Nucleus
of compressibility and lubrication of articular cartilaginous Mitochondria
surfaces of his knee joint. Loss of which of the following Golgi apparatus
extracellular matrix components has most likely occurred in this Endoplasmic reticulum
man?
Hyaluronan 21 A scientist desires to wipe out malaria-carrying mosquitoes
Fibronectin on a genetic level. He attempts to alter Anopheles mosquito cell
Elastin lines using the CRISPER/Cas9 system. Which of the following
Laminin is the most accurate role of Cas9 in this setting?
Integrin Amplifies the target DNA
Recognizes target DNA via guide RNA
16 The extracellular matrix and the cytoskeleton communicate Deletes the target DNA
across the cell membrane through which of the following Rejoins the cleaved target DNA
structures? Recognizes target DNA thru specific protein domains
Proteoglycans
Cadherins 22 Which of the following cells is most likely to die by apoptosis?
Intermediate filaments A cancer cell that overexpresses Bid
Integrins A leukemic cell with Apaf-1 protein deficiency
Microtubules A mutant cell that does not express FADD
A tumor cell that lacks expression of Smac/DIABLO
17 A cell infected with human herpesvirus 8 that produces FLIPs

A 27-year-old woman with a history of epigastric pain undergoes


an upper gastrointestinal endoscopy with biopsy. Endoscopy 23 Ultrastructural examination of a cell finds abundant smooth
finds gastric folds thickened and erythematous. Microscopic endoplasmic reticulum, numerous droplets, large mitochondria
examination reveals within the lamina propria clusters of with tubulovesicular cristae, and small dense bodies. Which of
mononuclear round cells with eccentric nuclei. Which of the the following physiological functions is most
following can be found in abundance in the mononuclear cells? characteristic of a cell with this ultrastructure?
Glycogen Motility
Cholesterol Mucus secretion
Immunoglobulin Steroid synthesis
Lipofuscin Zymogen granule production
Hemosiderin Glycogen synthesis

18 In a study, a type of skin cancer cell was found to synthesize 24 A research study evaluates a lung tumor in which a gene for
large amounts of ribosomes. Which component in this telomerase is
micrograph would be prominent in the identified in the DNA of the cancer cells. Which of the following
cancer cell? abilities would
B this gene provide the cancer cells with?
A (1 Point)
C Invade thru the basement membrane
E Invade thru the wall of a blood vessel
D Divide for an indefinite length of time
Implant in organs to form in metastatic lesions
19 A student takes on a project that involves the inheritance Increased rate of mitotic activity
pattern of coat colors in rabbits. She has black and brown
rabbits. The mating of two black rabbits yields black offspring. 25
Likewise, the mating of two brown rabbits yields brown offspring.
When black male rabbit mates with a brown female rabbit, all the A pathologist receives a salivary gland specimen for evaluation.
male offspring are brown, but the female offspring have patches For academic
of brown and black. When the brown male rabbit mates with a purposes, the pathologist uses an immunohistochemical stain
black female rabbit, all the male offspring are black, but the for vimentin on the
female offspring have patches of brown and black. The student specimen. Which of the following histologic structures is
thinks the gene for fur color is X-linked. Which of the following expected to be
forms of RNA is most likely to be involved in the patchy color immunoreactive with this stain?
pattern of the female offspring? (1 Point)
Mucous acini
Striated ducts Tyrosine contributes to the biochemical process leading to which
Serous acini of the
Nerve bundles following?
Fibrous stromal connective tissue (1 Point)
Blackish discoloration of the lung hilar nodes of a deceased city
26 dweller
Blue to black discoloration of the ear cartilage of a patient with
Two family members are found to have a genetic predisposition alkaptonuria
to develop skin Brownish discoloration of the heart of a deceased elderly patient
cancers at a very early age. Further work-up finds the loss of Tanning of the skin of a beachgoer who forgot to wear sunblock
p16 as the Darkening of the skin of a patient with chronic thalassemia
underlying cause. The result is a hyperphosphorylated Rb 31
protein. This
hyperphosphorylated protein promotes transcription needed for Which of the following is not a function associated with the
the transition smooth endoplasmic
from: reticulum?
(1 Point) (1 Point)
G0 to G1 Production of glucocorticoids by the zona fasciculata of the
G1 to S adrenal cortex
S to G2 Production of testosterone by Leydig cells
G2 to M Production of intramembrane proteins
M to G1 Drug detoxification by hepatocytes
Sequestering of calcium in skeletal muscle
27
32
A pathologist describes a hemorrhagic red lesion in one of the
organs he A neonate is born with “webbed fingers” (syndactyly) in both
examined during autopsy. Microscopically, he finds the lesioned hands. Which of the
organ to be following were most likely decreased in cells in the interdigital
infarcted. Which of the following is the most likely source of this region of the
tissue sample? developing hands of this baby?
(1 Point) (1 Point)
Kidney Cell swelling
Spleen DNA degradation by endonucleases
Heart Random DNA degradation
Testis Bcl-2
Lung Inflammation

28 33

A 55-year-old man develops myocardial infarction. He was given A pharmaceutical company attempts to develop a new agent
thrombolytic that stops leukemia
therapy, and his serum creatine kinase level went up. Which of
the following cells from dividing. The drug is found to be a potent inactivator
tissue events most likely occurred in the patient’s myocardium of chromatin-
after thrombolytic modifying activity that upregulates the expression of a cluster of
therapy? oncogenes in
(1 Point)
Reperfusion injury the leukemia cells. Which of these activities is most likely to be
Myofiber atrophy stimulated by the
Cellular regeneration enzyme target of this drug?
Drug toxicity (1 Point)
Increase in the synthesis of creatine kinase Methylation of cytosine bases in DNA
29 Binding to histone H1 to nucleosomes
Deacetylation of core histone H4
A cellular mutation results in a misfolded protein. This protein Deamination of cytosine bases in DNA
remains within the Acetylation of core histones
cell and is not excreted. Which one of the following cytoplasmic
enzymes is most 34
likely to activate?
(1 Point) At 40 weeks gestation, a 20-year-old woman delivers a healthy
Glutathione peroxidase infant and begins
Telomerase nursing postpartum. Her breast shows bilateral enlargement.
Caspases The receptor for the
Ribonuclease hormonal stimuli that accounted for these breast findings
NADPH oxidase (1 Point)
is associated with a trimeric G protein
30 is situated intracellularly and has a DNA-binding domain
can auto-phosphorylate and activate other proteins Euchromatin
is associated with a seven-helix transmembrane domain Centriole
belongs to the same receptor class as glucagon receptors
39
35
Which one of the following is the critical initiator caspase of the
Tumor cells from a leukemia patient have been analyzed to mitochondrial
determine which pathway of apoptosis?
gene is involved in the transformation. After partial sequencing (1 Point)
of the gene, the Caspase-8
predicted gene product is found to be a growth factor receptor. Caspase-10
This receptor is Caspase-9
most likely to be characterized by a /an: Caspase-3
(1 Point) Caspase-6
A seven-helix transmembrane domain
An intrinsic guanylate cyclase activity 40
A tyrosine kinase domain receptor
A helix-turn-helix transmembrane domain receptor A 70-year-old man with a history of difficulty with urination
An intracellular receptor with a zinc finger domain undergoes
36 transurethral resection of the prostate. The surgical pathology
laboratory receives
A patient undergoes total thyroidectomy due to thyroid cancer. the specimen and the tissue findings reveal nodules of glands
Microscopic with intervening
examination of thyroid specimen reveals a tumor with papillary stroma. Which of the following processes has most likely
architecture. occurred in the
Within one of the papillae, a small, round, laminated structure is prostate?
noted. Which of (1 Point)
the following is the basic defect producing this abnormal Apoptosis
structure? Dysplasia
(1 Point) Hypertrophy
Dystrophic calcification Hyperplasia
Bacterial infection Metaplasia
Viral infection
Metastatic calcification 41
Enzymatic necrosis
In a study, adult human fibroblasts are reprogrammed into stem
37 cells using the
transcription factors encoded by the SOX2 and c-MYC genes.
A 32-year-old man has had epigastric discomfort and burning Under appropriate
sensation in his conditions, the experimental cells generated all three primary
chest for several months. He undergoes upper gastrointestinal germ layers. Into
endoscopy, and a which of the following kinds of stem cells have these fibroblasts
biopsy is obtained from the epithelial lining of the lower third of been
the esophagus. transformed?
A microscopic exam shows intestinal-type epithelium. Which of (1 Point)
the following Pluripotent
statements is true regarding this case? Mesenchymal
(1 Point) Epithelial
The patient has an increased risk of developing squamous Lineage-committed
carcinoma if left unchecked for 2-3 years. Embryonic
The changes that occurred in the esophageal epithelium are
characterized by loss of cellular 42
specialization.
The change that occurred in the esophageal epithelium is a A liver biopsy taken from a 45-year-old woman with a PiZZ
result of gene activation due to the genotype reveals
acidic pH. PAS-positive globules within the periportal hepatocytes. Which
A diagnosis of reflux can be made based on the biopsy findings. of the following
The process that occurred in the esophageal epithelium is due molecular mechanisms is most likely responsible for this liver
to loss of intracellular communication. finding?
(1 Point)
38 Inability to metabolize AAT in Kupffer cells
Impaired disassociation of AAT from chaperones
Which one of the following structures is not part of a human Decreased catabolism of AAT in lysosomes
chromosome in any Retained misfolded alpha1-antitrypsin (AAT) in the endoplasmic
phase of the cell cycle? reticulum
(1 Point) Excessive hepatic synthesis of AAT
Centromere
Histone 43
Nucleosome
A 90-year-old man with severe dementia has been bedridden for
five years. A 45-year-old man is diagnosed with aggressive lung cancer. He
Physical exam finds no decubitus ulcers on his back, buttocks, dies of
or lower widespread metastasis despite chemotherapy. Autopsy reveals
extremities; however, his upper and lower extremities are tumors in many
reduced in organs. Microscopic examination finds some shrunken and
circumference, the lower extremities more so than the upper deeply eosinophilic
extremities, with tumor cells. Their nuclei show condensed aggregates of
apparent significant loss of muscle mass. Which of the following chromatin under the
combinations of nuclear membrane. The process affecting the tumor cells here
molecular mediators is most responsible for the decreased size is most likely
of the skeletal triggered by the release of which of the following?
muscles? (1 Point)
(1 Point) Phospholipase
12-Lipooxygenase-Lipoxin A4 Lipofuscin
Cyclooxygenase-Thromboxane A2 Catalase
p53-Bax BCL-2
Ubiquitin-proteosome Cytochrome c
Plasmin-C3a
48
44
An autopsy is done on a 70-year-old patient with a history of
A 55-year-old nonsmoker with normal lung function is killed in a sudden death.
hit-and-run Multiple tissue sites are sampled for histopathologic analysis.
accident. At autopsy, the pleural surfaces of his lungs have a Microscopic
reticular black examination shows noncrystalline amorphous deposits of
appearance. Which one of the following accounts for this gross calcium salts in the
appearance? gastric mucosa, renal interstitium, and alveolar walls of the
(1 Point) lungs. Which of the
Melanin pigments following processes accounts for the appearance of these
Lipochrome pigments calcium deposits?
Anthracotic pigments (1 Point)
Hemosiderin-laden macrophages Hypercalcemia
Lung infarction Dystrophic calcification
Normal aging process
45 Apoptosis
Excessive calcium ingestion
A 30-year-old woman is diagnosed with viral hepatitis. Part of
her work-up 49
includes a liver biopsy which reveals scattered, eosinophilic
cells, associated with Which of the following factors is a potent inducer of
sparse inflammation. Of the following cellular components, angiogenesis?
which one most (1 Point)
directly contributed to this almost lack of an inflammatory Platelet-derived growth factor
reaction? IL-1
(1 Point) Epidermal growth factor
Phagosome oxidase Endostatin
Phosphatidylinositol Basic fibroblast growth factor
Phosphatidylserine
Phospholipase 50
CD95
A mother brings a 3-year-old Caucasian boy with a history of
46 recurrent lung
infection and chronic diarrhea. Further investigation reveals a
Laboratory testing of a 30-year-old woman with a facial “butterfly mutation in a
rash” reveals protein that normally controls the passage of salt across cell
high titers of antinuclear autoantibodies, Smith antigen, and anti- membranes. As a
nucleosome result, the salt in his sweat is abnormally high, and mucus
antibodies in the serum. Which of the following is most likely to normally made in with
be directly lungs, pancreas, and digestive system cannot retain water. The
affected by the disruption of nucleosomes in this patient? mutated protein
(1 Point) involved is classified as a
Forming pores for bilateral nuclear-to-cytoplasmic transport (1 Point)
Transcribing DNA Receptor
Packaging of genetic material in a condensed form Linker
Holding together adjacent chromatids Channel
Forming the nuclear matrix Enzyme
Structural
47 Submit
Liver Cirrhosis
1.Which of the following patients will most likely develop
significant edema? 9.The ultimate goal of the complement system is to induce:
(1 Point) (1 Point)
46/F who underwent extensive surgery and chemotherapy for Dysplasia
breast cancer Opsonization
50/M who had a mass excised from his big toe Metaplasia
12/M who had a tooth removed Lysis
23/M who exercises once a day
10.Congestion:
2.In chronic passive congestion of the liver, what morphologic (1 Point)
findings can NOT Is a passive process
be seen? Consider that the blood supply of a hepatic lobule is Characterized by cyanotic tissues
the central Does not occur locally
artery. Due to impaired outflow of arterial blood to a tissue
(1 Point)
Necrosis of the hepatocytes at the center of the hepatic lobule 11.The most common type of collagen is:
Necrosis of the hepatocytes at the periphery of the hepatic (1 Point)
lobule and sparing of the center Type IV
Hemorrhage Type I
Hemosiderin laden macrophages Type III
Type II
3.Hemosiderin laden macrophages in Chronically Congested
lungs are called 12.What is the difference between regeneration and healing?
(1 Point) (1 Point)
Hemosiderin cells never ends in scar
Lung failure cells Fibrosis rarely occurs
Cells of Zahn Healing requires an event that damages it in the first place
Heart failure cells Fibrosis occurs in proportion to the damage in the intracellular
matrix
4.The edema fluid that accumulates due to increased
hydrostatic pressure or 13.all three complement pathways lead to the splitting of C3 into
reduced intravascular colloid pressure: C3a and C3b. Which of the following is true of these two
(1 Point) products?
Has a specific gravity greater than 1.020 (1 Point)
Is a protein rich transudate further movement of the complement system leads to the
Is a protein poor exudate creation of the Membrane Attack
Has a specific gravity less than 1.012 Complex, composed of multiple C12 molecules
C3b is attached to the cell where complement is activated
5.one of the actions of Bradykinin on the vascular system is: binding of C3b leads to the formation of C7 convertase
(1 Point) C3a binds with C2 to create C123
increase vascular permeability
induces relaxation of smooth muscle 14.Which of the following organs/body parts is likely to suffer a
serves as an analgesic when injected intradermally white infarct?
constricts blood vessels (1 Point)
Liver
6.The acute inflammatory process may finish or terminate in one Stomach
of the following Lungs
ways: Spleen
(1 Point)
progress to chronic inflammation 15.Which of the following patients would you expect there to be
resolve, but produce a scar impaired wound
complete resolution healing?
all of the given choices are correct (1 Point)
A patient with aplastic anemia
7.Subcutaneous Edema: A patient with an active tumor
Accumulates preferentially where hydrostatic pressures are All of the given choices are correct
lowest A morbidly obese patient
Least pronounced in the legs while standing
Seen mostly in the arms when standing 16.A body is taken in for autopsy after suffering a pulmonary
Accumulates preferentially in areas most distant from the feet embolism. You
dissect the respiratory vasculature to see which clot was the
8.In terms of the type of pathophysiologic cause of edema, most likely cause.
which of these four Which of the following features would you consider as
causes is different from the others? suspicious for a
(1 Point) premortem clot?
Nephrotic syndrome (1 Point)
Malnutrition No lines of Zahn
Constrictive Pericarditis non-friable
adherent to the vessel wall
Resembles chicken fat 24.In healing by first intention, which of the following events can
be seen in
17.a person comes to your clinic for persistent cough, weight around 24 hours after the injury?
loss and night peak of neovascularization
sweats. Examination of this person's sputum reveals positive neutrophils are seen at the incision margin
acid fast bacteria. "blanching" begins in the wound
What kind of inflammatory process is going on in this patient's rapid activation of coagulation pathways
lungs?
foreign body type inflammation 25.Lipoxins function to do the following:
the lungs are fine, the actual lesion is in the trachea vasodilation
acute suppurative inflammation in the periapical lymph nodes facilitate chemotaxis
necrotizing granulomatous inflammation synergize with the actions of leukotrienes
vasoconstriction
18.A woman suddenly undergoes DIC after prolonged sepsis.
What lab or 26.A patient undergoes nephrectomy due to a significant
histopath findings can you see in this patient? staghorn calculus. a
(1 Point) year post of, you notice that the remaining, otherwise normal
Crescentic glomerulonephritis with anti-GBM kidney has
Cerebral atrophy increased in size. This is an example of:
Hemosiderin laden macrophages in the heart Dysplastic transformation
fibrin clots in the glomeruli Healing and fibrosis
Hydronephrosis
19.post operation, a patient develops ulcers around the wound. Compensatory growth
What is a
possible mechanism for this ulceration? 27.You suture a relatively simple wound in the ER. A carefully
(1 Point) sutured wound has
adequate granulation tissue formation approximately what percent strength compared to normal skin?
incomplete activation of platelet delta granules 40%
inadequate vascularization during healing 99%
fibrosis and keloid formation 70%
20%
20.Hemorrhages in the skin that are 0.7 cm in diameter are
called 28.of the following, which is clinically the MOST important
(1 Point) causes of healing
Ecchymoses delay?
Purpura (1 Point)
Bruises small foreign bodies
Petechiae Infection
fruits only diet
21.A 46/F has pitting edema of the legs. Which of the following location of the injury in the antecubital space
conditions is she
likely to have? 29.Which of the following patients does NOT have a higher risk
(1 Point) of DVT?
Arterial insufficiency Patient with third degree burns over 90% of the body
Asbestosis none of the above choices
Nephrotic Syndrome a post op patient immobilized on both legs
Rhabdomyolysis a person going on a plane trip from Manila to New York City

22.Peppa was a 23/F who was involved in a motorcycle 30.Hyperemia:


accident, breaking her left (1 Point)
femur. 24 hours after hospitalization, she complained of difficulty Is a passive process
breathing. She Has tissues that are engorged with deoxygenated blood
soon went into a coma and died. Autopsy findings are likely to Is characterized by decreased blood flow to tissues
find what kind of Occurs at sites of inflammation
embolus as a cause of death?
(1 Point) 31.A man was attacked with a hacksaw and was admitted to the
Amniotic Fluid ER. you notice that the wound is jagged and upon trying to repair
Gas it, there are chunks missing and the edges of the wound are not
Thromboemboli lined up. What do you expect during the healing of this wound?
Fat More epithelialization is expected
There is just as much granulation tissue compared to a wound
23.What would you normally expect with an immunocompetent of first intention
patient with an This is a wound of first intention.
ongoing acute inflammatory process? There is less fibrosis
increased ESR
negligible CRP 32.A person is bitten by a poisonous cobra and is brought in to
decreased body temperature and chills your clinic. He is treated with an antivenom. Which complement
10% Neutrophils, WBC 2.0
system pathway is likely triggered in this case in order to cleave
C3?
lectin pathway
classical pathway
none of the above
alternative pathway

33.In which of the following patients would you look out for an
embolic effect?
A victim of a car accident with multiple fractures
All of the above
A diver who surfaces too quickly
A patient with a long and difficult labor/delivery
34.The gross morphologic appearance of a liver that has
undergone chronic
passive congestion is called:
(1 Point)
Apricot liver
Cashew liver
Walnut liver
Nutmeg liver

35.Edema:
Is a process where fluid moves outward from the extravascular
to the intravascular space.
Is a process where fluid moves inward from the extravascular to
the intravascular space.
Is a process where fluid moves inward from the intravascular to
the extravascular space.
Is a process where fluid moves outward from the intravascular
to the extravascular space.

36.A 65/M suffering from liver problems has a bloated peritoneal


cavity. Peritoneal collections of edema fluid is called:
Anasarca
Ascites
Peritonitis
Peritoneal hydritis

37.A woman has an inflammatory reaction to her breast implants


a few months post op. No abscess formation or infection is seen.
Histopathologic examination of the breast tissue reveals what
type of response?
Foreign body granuloma
Extensive exudates
Teuton giant cells and a prominent mastocytosis
Caseation necrosis

38.Fibrinous exudates develop when:


there is a large vascular leak
there is a local anticoagulant stimulus
there is an ongoing infection of mycobacteria that causes
tuberculosis
a limb is amputated

39.During the formation of a platelet plug, this strengthens and


hardens a clot, also contracting the plug:
Thromboxane A
Hyaluronin
Fibrin
Prostaglandin

40.the following cells are involved in the acute inflammatory


response EXCEPT:
Neutrophilic Granulocyte
plasma cell
Neutrophil
Polymorphonuclear cell

You might also like